NBDE 2012 ALL REMEMBERED QUESTIONS

DAY 1

 

Know Patient Mgmt part of Mosby…its tricky too!

Know how to diagnose between periodontal vs periapical/irreversible vs reversible, pulpotomy vs pulpectomy…as long as you know the key symptoms I think you will do fine.

 

MAKE SURE YOU READ CAREFULLY CUZ SOME OF THE QUESTIONS ARE WORDED FUNNY (to me at least?!)

 

Pics I had to ID were:

AOT (Adenomatoid Odontogenic Tumor)=the stem had a 19 yr old female & xray was in the ant maxilla bet LI & K9 (most common presentation) Mosby pg. 119 Epithelial odontogenic tumor. Zellballen(whirling pseudocyst). Uniolocular RL appearance and snow flakes. Impacted Canine. Majority maxilla, females, anterior jaws and over crown of impacted tooth Differntial Diagnosis:  Lateral Periodontal Cyst Mosby pg 117 Unilocular/Mutilocular RL in lateral perio membrane.  Most found in mandibular PM Region tooth is vital. Gingival cyst of adult is soft tissure counterpart of this lesion

 

Epulis Fissuratum=edentulous ridge with excess tissue  Mosby pg 322 hyperplastic tissue reaction caused by ill-fitting or over extended flange in denture.  Tx. Adjust denture border and use tissue conditioner,surgery is indicated if inadequate response

 

Stafne’s defect (it was called Salivary gland inclusion)=typical pan with lesion under IA canal Mosby pg. 104 RL of mandible due to invagination of the lingual surface of jaw located in the posterior lingual of mandible below mandibular canal. Impinges of mandibular nerve. Also called Static Bone Cyst

 

AI( Amelogensis Imperfecta)=PA of teeth with open contacts & no diff between enamel and dentin. 3 types.

1.) Hypoplastic – inadequate deposition of emamel matrix

2.) Hypomaturation –defect in maturation of enamel crystal structure. Abnormal hardness. Less Radiodense than dentin. Softer than normal

3.) Hypocalcification- normal enamel matrix and no significant calcification.  Affects enamel only without systemic disorder.  NOTE:  Radiographs are normal may see taurodontism on occasion and cant diagnose w/ x-rays alone (ORAL PATH NOTES) Mosby pg. 175 associated with BELL STAGE (Histodifferentiation)

Note:

 

Lower border of the nose=ant PA of missing 8 & 9 and pointed to it Mosby pg 135

Lateral wall of the orbit=half of a pan of an edentulous maxilla (idk if that was right but that’s what my answer was) Mosby pg 141

 

Pyogenic granuloma ( pg 112 mosby.AKA Pregnancy Tumor (1st trimester) due to unbalanced hormones or Eruptive hemiangioma or granulation tissue-type hemiangioma – according to wiki , Not true tumor, local irritant that causes overgrowth of gingiva tissue, Color ranges from pink, red, purple, can be smooth lobulated, often seen on the anterior nasal septum, occurs on gingiva 75%, maxillary > mand. Anterior region > posterior. Can also be found on the lips, tongue, and cheek.

The 3P’s:  all have same ccl and different histo 1.) Pyogenic Granuloma 2.) Peripheral Giant Cell Granuloma- females 40-50, serum calcium test performed associated with MNGC’s. Sessile or pedunculated 3.) Peripheral Odontogenic Fibroma – bone or cementum calcifications.

 

Fibrous dysplasia  Pg. 120 Mosby- CCL- More common in maxilla. Affects Children, Radiographic appearance- diffuse opacity (ground glass- frosted beer mug) Oral Path notes- Histology- Chinese Characters. 2 types monostotic and polystotic. Associated with 2 syndromes 1.) McCune Albrights ( café au lait pigmentation, endocrinopathies (percious puberty, pituitary adenoma, hyperthyroidism, polystotic) 2.) Jaffe-Lichtenstein (no endocrinopathies) Pt. can have facial swelling and expansile of jaws

 

Gingivitis scale is what type of system? (I put ordinal because it had mild=1, moderate=2, severe=3 in the stem)

 

 

What does the W on the rubber damn clamp stand for? Wingless (luckily I saw this before & found the answer in our oper txtbk..lol)

 

Pt lip swells up right after impressions, what caused it? Angioedema, anaphylaxis (Angioedema may be caused by an allergic reaction. During the reaction, histamine and other chemicals are released into the bloodstream. The body releases histamine when the immune system detects a foreign substance called an allergen. The main symptom is swelling below the skin surface. The following may cause it: animal dander (scales of shed skin, exposure to water, sunlight, cold or heat, foods (berries, shellfish, fish, nuts, eggs, milk, insect bites, medications (penicillin, sulfa drugs, NSAIDS, blood pressure medicine (ACE inhibitors) . Hereditary angioedema is rare but problem with the immune system that is passed down through family. It causes swelling in face, airways, and abdominal cramping– resource http://www.ncbi.nlm.nih.gov/pubmedhealth/PMH0001849/  )

 

-          Older female w/ widening of PDL and resorption of mand? Scleroderma (it said diffuse systemic sclerosis) Mosby pg 109 An autoimmunie multiorgan disease of adults especially women.  Fibrous of tissues leads to organ dysfunction. Oral  changes include restriction of orifice, uniform widening of periodontal membrane, and bony resorption of mandibular margin of mandibular ramus (best seen on PAN). CREST Syndrome- Calcinosis cutis ( calcium salt deposit beneath the skin), Raynauds phenomenon ( temp change from stress tips of fingers turn blue can be tx with nifedipine, amlodipine, dilitiazem, felodipine, nicardipine) Eshophageal Dysfunction, Sclerodactyly, Telangiectasis. Diffuse systemic sclerosis is rapidly progressing and affects large area of skin and one or more internal organs (kidneys, esophagus, heart,lungs)

 

-          Whats the least common congenitally missing tooth? (3rd, Man 2nd, LI, K9)

Mosby pg 193. Most common congenitally missing tooth is 3rd Molars,  mand. 2nd Pm, Max. LI, and then Max. 2nd PM

 

-          S, z, ch sounds, do what to the teeth?(I put bring them the closest together)

o    Sibilant/linguoalveolar sounds (s,z,sh,ch,j)–tip of tongue contacts anterior palate/or lingual surfaces of teeth.  Determine vertical overlap &length of anterior teeth.

o    Mosby pg 233-324 Fricatives or labiodental sounds f, v, ph are made btw max. incsiorss contacting the wet/dry lip line of mandibular lip. These sounds help to determine the position f the incisal edges of maxillary anterior teeth

o    Linguodental sounds  are those using tip of tongue slight between max and mand teeth to say this, that. Those.  If the tip of tongue is not visible the teeth most likely too anterior EXCEPT in Class II malocclusion or excessive vertical overlap where the tongue extends too far out and teeth too far lingual

o    The B, P, M sounds are made by contact of the lips. 

 

-          Pt wearing a denture for 19 yrs, wants a new one, you notice a 6x3cm white lesion, what do you do? Biopsy, refer to head & neck cancer, adjust denture (IDK but that’s what I put) Its incisional biopsy bc the lesion is larger than 1 cm. if it was less than 1 cm you would do excision. Mosby pg 95: Different types are 1.) cytology smear 2.) aspiration 3.) excision 4.) incision

 

-          Pt has denture for 14 yrs, you notice red raised papules on palate, what is it? (Inflammatory hyperplasia of palate, nicotinic stomatis – may have red and gray spots))

o    IPH-usual cause under ill- fitting denture, denture movement irritation and/or accumulation food debris.  Presents as painless, firm pink and red nodular proliferations of mucosa.  Hard palate usu., may involve residual ridges.  Not completely reversible but can regress w/ smllr papilla and tx (remv denture, soft relines, good oral hygiene, and nystatin therapy. Pt. needs to soak denture in 50% water 50% bleach

-          Pt is on Propanolol for HTN, you give 10mg/ml EPI, what happens? (inc bronchiodilation, inc HR, inc BP…I think I chose inc BP) Propanolol beta blocker. (b1). Epi is a vasoconstrictor causes slight elevation.

 

-          What drug doesn’t cause gingival hyperplasia? (Dilantin –phenytonion-anticonvulsant, cyclosporine( gin graft/ neural)- prevent transplant rejection nifedipine –(Procardia)- calcium channel blockers, Verampil- calcium channel blocker,  a random drug)

 

-          Where to place the implant in relation to the adjacent teeth? (asked me this 2x in 2 diff ways) I put below adjacent CEJ both times (Emergence Profile- 2-3 mm apical to the adjacent tooth CEJ)

 

-          What the most effective brushing technique? (stillman, sulcular, and two other choices….I chose sulcular bcuz I thought it was the bass technique) (Correct The Bass Method is also called the Sulcular Vibration technique.. named after Dr. Charles Bass. This techniques is when the toothbrush bristles are placed at the gingival margin at 45 degree angle t o the tooth allowing bristles to extend into gingival sulcus when pressure applied. Mosby pg 268

 

-          If you have light ortho force, bone resorption is where in relation to the force? (undermined, direct, indirect) Mosby pg. 156 Light forces causes smooth continuous movement without formation of hyalinized zone in surrounding PDL. The teeth start to move earlier in a more physiological way than with heavy.  Heavy force actually delays tooth movement by causing a lag period after the initial movement of tooth within PDL.  Undermining Resorption occurs within the alveolar bone in the marrow spaces and moves toward the PDL area.  It is associated with heavy force. Appearance of osteoclastic cells in bone marrow spaces is the first indication of undermining resorption.

 

-          What cells are found in the established lesion of gingivitis? (Plasma cells, macrophages, lymphocytes, and another one) Mosby pg. 248

1.       Initial Lesion- 2-4 days, PMN’s, Blood vessels is vascular dilation, collagen  perivascular loss, and CCL findings gingival fluid flow

2.       Early Lesion – 4- 7 days ,Lymphocytes, Vascular proliferation, collage increased loss around infiltrate and CCL findings erythema and bleeding upon probing and

3.       Established Lesion- 14-21 days, Plasma Cells, vascular proliferation and blood stasis, continued loss in collagen and CCL findings is changes in color, surface, texture Advanced Stage characterisitics of stage III move into periodontal ligament space and create periodontitis.

 

-          List the orders of Perio therapy…said it like Emergency, plaque control, reeval, surgical, maintanence…shit I cant remember but you will recognize it (Mosby pg 252.

1)       Preliminary Phase (tx of emergencies, ext of hopeless teeth)

2)       NONSURGICAL PHASE (phase 1 therapy- plaque control, diet control, removal of calculus, excavation of caries, antimicrobial therapy, minor ortho, provisional splinting)

3)       SURGICAL PHASE (phase II perio including implant placement and endo therapy) RESTORATIVE PHASE (phase III final restorations, Fixed, RPD, evaluation of response to procedures periodontal examination)

4)       MAINTENANANCE PHASE (phase IV perio rechecking)

 

-          In trisomy 21, you will see all the following (LARRY?!...lol)EXCEPT? Rampant caries (you will see perio disease though) Trisomy 21=Down Syndrome ( no gingivitis, but periodontitis,  very loving patients who like to hug on you, Provetella Intermedia organisms(orange complex) are associated with these pts.

 

NOTE:  Prevetella Intermedia organism is associated in pregnancy and necrotizing perio diseases and down syndrome. 

 

RED complex organisms ( P. gingivalis, T.forsytheia, T. denticola) “Dentists for Red Gingivitis!” (so we can get paid!)

Orange Complex (P. Intermedia, F. bacterium, Camphylobaceter) “Camping In back (woods) with Orange fire”

 

-          What symptom will a person with trisomy 21 have? Small midface but worded diff

 

-          What systemic antifungal will you give a person w/ oral candidasis w/ HIV? (Fluconazole was the only system one listed), or Ketoconazole or Amphotercin B. , Caspofungin. Note: regular oral candidiasis infections are treated with nystatin rinses or clomatrizole troches. DOC for HIV pts with candidi is FLUCONAZOLE!

 

-          Woman allergic to pen, and needs extractions, what to give before? (600mg clinda 1 hr before…the other choices were the right drugs but wrong doses)

SBE Prophylaxis Mosby pg 308

Look at cephalosporin  chart Mosby pg 307

-          8 month pregnant lady gets extractions, flap, and osseous contouring?! (something that was gonna cause some post op pain)what can you give? (acetaminophen 325mg,  Tylenol #3, and 2 others that seemed like NO-NOs…idk but I chose Tylenol #3 dammit cuz it seemed like the right thing to do..lol) Note: Pregnant women can have tylenolol, propofol, codeine, propoxyphene.  CANNOT GIVE warfarin, NSAIDS, methotrexate, merpidine, nitrous, barbituates.  Tylenol #3 contains 300mg acetaminophen and 30 mg of codeine. Tylenol #4 contains  300 mg acetaminophen and 60 mg of codeine.

 

-          What causes tardive dyskensia? (phenothiazine) Tardive dykensia is involuntary movement of the muscles.  Mosby pg 281  Phenothiazine is an antipsychotic. Indications schizophrenia other types of psychosis, tourette’s syndrome, hungtington’s chorea, and other disorders like obsessive compulsive disorders.  Adverse motor effects of antipsychotic drugs acute dystonias, akathisia, parkinosonism, perioral tremor, malignant syndrome and tardive dyskinesia or extrapyramidal.

 

-          What causes extrapyramidal stimulation? (antipsychotic-phenothiazine)

 

How do sulfonylureas oral hypoglycemics work? Stimulate the release of insulin from “GGGotta gave insulin!” pancreas Mosby pg 303   Glyburide, Glipizide, and Glimepiride. Know Insulin table on pg 303

 

-          A person on chronic glucocorticoids, you see all the side effects except? (Hypoglycemic, weight gain, osteoporosis, and 1 more) Pg 303 Mosby  Adverse Effects insomnia, agitation, infections, hypertension, atherosclerosis,  skin and mucosal atrophy, negative calcium balance (osteoporosis) muscle wasting, obesity, glucose intolerance, peptic ulcers, and cataracts

 

-          Which symptom with you find with corticosteroid treatment? Osteoporosis

 

-          The main effect that causes osteoradionecrosis? Something about effects the vascular supply

 

 

 

-          Pt on IV bisphosphonates and has carious and non restorable teeth, what is the best treatment? (I put the answer that said discontinue for 3 months bcuz they didn’t have one about talking to the physician)  IV Bisphosphonates ( Zolendroic Acid (Zomate), Pamidronate(Areida)) Oral Drugs (Fosamax (aldrenote), Actonel(Risedronate) and Boniva (Ibndronate). Bisphosphonates can cause osteoradionecrosis of the Jaw so treat with hyperbaric oxygen during dentoalveolar surgery. Note: IV bisphosphonates have a half life of like 14 YEARS!!! If you need to do anything, you just cut the crown of at the gingival and do RCT. Do not EXT unless pt is well aware of the risks…even then, be careful.

 

-          Pt on takes a aspirin, how long does it inhibit the platelet activity? (1 hr, 1 week, 24 hrs, 1 month…IDK) Board busters pg 176 discontinuation of aspirin for 5 to 7 days allows normal clotting time to reapprear due to synthesis of new platelets. Depending on the frequency and dosage depends on how long it takes to inhibit the platelet activity

 

-          What does Warfarin inhibit? The vitamin K dependent clotting factors Clotting factors are II, VII,IX, X.  Warfarin and Coumadin are vitamin K antagonist. 

 

-          Pt had a injection of 4% PRILOCAINE, and had cyanosis of lips and area of injections, what happened? (I chose METHEMOGLOBINEMIA…cuz that was always the damn answer when it came to prilocaine)

 

-          A drug NO INTRINSIC activity, what is it? (agonist, antagonist) Mosby pg 271

 

-          Which structure is involved in a Leforte I fracture? Maxillary sinus Mosby pg 83

 

-          What mandibular fracture will cause parasthesia? (Symphisis, coronoid process, condyle, body) Most common fractures is condyle, angle, symphisis, body

-          The advantage over distraction osteogenesis vs split osteotomies…(you could probably pick the right answer after reading the choices) Mosby pg83-82-83. BC of Bone growth with distraction osteogenesis

 

-          What structure is similar to an epiphyseal plate? Synchondroses Mosby by 146 Sychrondoses become inactive.  Intersphenoid age 3, Spheno-ethmoid age 7, and spehno-occipial considerly.

 

-          The most common between cleft lip/palate, ectodermal dysplasia, AI, DI, and OI? Cleft palate higher in females and is most common in Native Americans and Cleft Lip is higher in males. Cleft Lip/Cleft Palate is more common in men and more unilateral and unilateral is more prevalent in women and bilateral more prevalent in men

 

-          ? Race more prone to perio disease in US? (Black male, White male, Asian female, native American female)

 

-          Around menarche, you will see what? (sexual maturation, dental maturation, something else, and more prone to perio…I chose it bcuz of the common time for LAP) Note P. intermedia organism more prevalent in pregnancy, puberty, necrotizing perio, and down syndrome patients “Prevetella got pregnant during puberty”

 

-          What stage of histology can you tell the diff in size and shape of tooth formation? (proliferation, histodifferentiation, morphodifferentiation…got my ass?! Look it up) Mosby page 175-176 answer is Morphodifferentiation (Bell Stage)- peg laterals, macrodontia. Proliferation (Cap Stage)-supernumerary teeth, anodontia, cysts, odontomas Histodifferentiation (Bell Stage)- AI, DI, OI Apposition- incomplete tissue formation-i.e. enamel hypoplasia and Calcification- localized infection, trauma, fluorosis, tetracycline staining.  So the order of tooth development.

  1. Initiation (Bud Stage) – absence of this is anodontia, excessive bud is supernumerary teeth.
  2. Proliferation (Cap)
  3. Histodifferentiation/Morphodifferenitation(Bell Stage)
  4.  Apposition
  5. Calcification

 

-          What is the internal connection (hex) between the implant and abutment designed for? I chose to prevent rotation of the tooth (sounds like antirotation to me)

 

9 yr old pt chipped ant porcelain veneer but wants it fixed not replaced, how do you prepare? (had to put in order….acid etch, silanate, bond resin, pumice…I guessed) Brush porcelain 1.etch onto the inside surface of the veneer with the small brush included with the etch. Brush up and down across the entire veneer to create a rough surface, which is what will allow the porcelain silane adhesive to adhere better. Allow the etch to dry for 10 minutes.

2.Brush porcelain silane onto the inside surface of the veneer with the brush included with the silane. Brush in a side-to-side motion across the entire veneer.

Place the veneer back on your tooth carefully, easing it slightly under your gum. If the veneer is fractured, squeeze the two sides together after the veneer is on your tooth to close the fracture.
Hold the veneer on the tooth with your fingers for a minimum of three minutes. Wait four hours before eating to ensure the veneer has bonded securely to the tooth.



-          Pt wanted veneers and his teeth bleached, what order would you do it? (I chose bleach, 2 week break, cut teeth cement veneers)

 

-          Ant PFM looks too opaque in the incisal 1/3, whats most likely the problem? (inadequate biplane reduction)

 

-          Whats the posterior palatal extent of a complete denture? (hamular notch, vibrating line, fovea palatine…I chose vibrating line bcuz it said posterior PALATAL extent) Mosby pg 324 The posterior limit extends to jxns of moveable and immovable tissue. This coincides with the line drawn through the hamular notches and approx. 2mm posterior to fovea palatine (vibrating line)

 

-          Why would you reduce the opposing dentition before doing a fixed bridge? Supraeruption

 

-          The difference between 245 and 330? 245 longer than 330 (its obvious)

 

 

-          Patient on Warfarin? What test do you order? INR ( International Normalized Ratio used to monitor the effectiveness of blood thinning drugs such as warfin.  Recommended valuses 2-3 or 2-4. PT (Prothrombin Time- 11-13 secs in normal person but 12- 18 in patients on warfin), PTT (Partial Prothrombin Time – 25-35 secs. In normal ppl. Test of hemophilia, bleeding time less than 9 minutes)

 

-          What is the minimum distance that you need between implants? 3mm (Note: 1mm btw implant and non-implant tooth)

 

-          What is the angle of the curette when doing SRP? (20-40, 45-90, 90-120)

 

-          Where primate space located? (had two columns, max=LI, K9, man=K9,1stM)

 

-          Pt has multiple osteomas, colonal polyps, and dermal cysts, what is the syndrome? Gardners –Mosby pg. 123 autosominal dominant, supernumery teeth. Intestinal polpys have a very high rate of malignant conversiton to colorectal carcinoma.

-          Whats the important factor in DFDB graft that gives it osteogenic properties?( BMP) (Note DFDB is osteoinductive and FDB is osteoconductive. BMP stands for Bone Morphogenetic protein)

 

-          What is not a factor sodium hypochlorite? Chelating agent(its non-chelating and organic)

 

-          Most common location for a siaolith? Submandibular duct

 

-          What syndrome will you see multiple OKCs? Gorlin or Nevoid Basil Cell Syndrome hereditary autosomal dominant multiple basal cell carcinoms of the skin, multiple OKS, bifid ribs, frontal –bossing, calcifications of falx cerebi, palmer and plantar pitting.

 

-          Pt has a new PFM place with overhang, what is the first symptom you will see? ( I just remember gingival inflammation and gingival recession)

 

-          Pt has probing depth of 5mm, the CEJ is 2mm coronal to the free gingival margin, what is the depth? 7mm…I hope?! Lol

 

-          How do you measure the periodontial attachment? (I took it as they were asking the clinical attachment level, a fixed, CEJ, point to the bottom of the pocket)

 

-          How does Zantac(Ranitidine) work? Inhibit gastric acid secretion

 

 

-          Which impression material is not used for doing cast impressions? (I was in between reversible & irreversible but I thought of DR Tyus in the review when he said the best impression material is Agar Agar, reversible, so I chose Irreversible)

 

-          Pt had a rough, white, pedunculated nodule on palate, what is it?(papilloma)

 

-          Why is a pulpotomy on a 6 yr olds perm 1st molar the treatment of choice?(choses were for apexification, physiological growth of root, to help 2nd molar erupt..i chose my answer bcuz it sound like apexogenesis and you wud still have vital pulp in roots)

 

-          What does primary tooth discolor after trauma?(diffusion of blood in dentin tubules, fibrosis of pulp, 2 more choices)

 

-          What type of occlusion precedes molar class II occlusion? Distal step class I is flush terminal plane, class II is distal step and distal step is least desirable and class III is mesial step

 

 

-          Stidor is seen in what? (bronchospasm, laryngospasm, and to other random choices)(High pitch wheezing that is inspiratory)  Tx with succinylcholine High pitch wheezing expiratory is asthma) Tx asthma with albuterol, theophylline or epinephrine.

-          Most common salivary gland tumor? Pleomorphic adenoma Mosby pg 115 Most common malignat salivary malignancy in minor and major glands is Mucoepidermoid Carcinoma.

 

-          What is the treatment of choice for a tooth that is showing inflammatory external resorption? Pulpectomy w/ Caoh2 mosby 24 via internet External inflammatory resorption and replacement resorption are complicating factors that may result from traumatic dental injuries when the tooth is luxated or avulsed and replanted. Resorption may, ultimately, result in loss of the tooth. However, with appropriate treatment, the prognosis for these teeth is greatly improved, with the possibility of preventing or arresting resorption. EXTERNAL ROOT resorption occurs more frequently than internal root resorption and is commonly misdiagnosed as internal root resorption. External root resorption is caused by an injury to the external root surface. After an injury such as concussion or subluxation, cementum can be damaged, resulting in a localized inflammatory response and area of resorption. In about two weeks, the periodontium and root surface should repair spontaneously, and in that case no treatment is needed. With severe injuries, such as intrusion or avulsion (especially when implantation is delayed more than 60 to 90 minutes), active external inflammation can persist and histologically there will be multinucleated osteoclasts resorbing the dentin of the root. Seven to ten days after the injury, it is recommended to treat the tooth endodontically by placing calcium hydroxide in the canals long-term and replacing the calcium hydroxide in one month and then at three-month intervals until the resorptive process ends. The high pH of the calcium hydroxide seems to permeate through the dentinal tubules thus killing bacteria and neutralizing endotoxin, which stimulates inflammation. If bacteria are thought to originate in the sulcus of the tooth (totally external), a vitality test will respond positively, but in cases where infected pulp causes external root resorption usually in the apical or lateral aspects, a vitality test can be negative. 
     The cause of internal root resorption is unclear, but trauma and the extreme heat produced when using a high-speed drill without water have been suggested. Histologically, there is normal pulp tissue transforming into granulation tissue with giant cells resorbing the dentinal wall, and resorption will only occur if the odontoblastic layer and predentin are lost or altered. Internal root resorption usually contains some vital pulp and gives a positive vitality test; however, since necrotic pulp tissue is usually found coronal to the active resorbing cells which are more apical, the tooth can sometimes test negative. Internal root resorption resolves with root canal treatment because the resorbing cells will no longer have the blood supply to survive. In cases where internal root resorption causes buccal or lingual perforation, mineral trioxide aggregate (MTA) can be used to repair the site.

-          You use a complimentary color on a pfm, what does it do? (decrease the value…blue is the color use to decease the value which is the complimentary of yellow)

-          Doing RCT on max 1st molar, what is the surface on the MB root that is most common for strip perforation? (mesial, distal, buccal, lingual)

-          The latency of radiation therapy is between? (therapy & film development, therapy & symptoms, 2 more) Mosby pg 130 so I think therapy and film development

-          Which has the greatest cariogenic potential? (sucrose, lactose, glucose, fructose)

-          Opiods cause vomiting but? Stimulation of the medullary chemoreceptor trigger zone

-          What syndrome has congentially absent teeth? Cleidocranial dysplasia, downs syndrome, ectodermal dysplasia (it was an obvious choice) Ectodermal dysplasia ppl have thin sparse hair, anodontia/oligodontia, and dry scaly skin

-          What is the best way to clean an interproximal space? (interproximal brush, toothpick, water irrigator)

-          How much do you take off of the middle 1/3 of the facial surface when preparing an anterior veneer? (.3, .5, .8, 1)

-          A needle tract infection following an IANB caused and infection in what space? (Sublingual, submandibular, temporal, pterygoid)

-          Trismus is usual caused by infection in what space? Masseteric

-          What muscle helps mold the lingual flange of the mandibular denure? Mylohiod, geniohyoid, masseter, ..) PMS-G (Palatoglossus, mylohyoid, genioglossus, and superior constrictor muscle) all contribute

-          When removing the internal oblique ride, what is the risk associated? …

-          Pt has a fracture of the right body of mandible, where should you also check for a fracture? Left condyle Concept whatever side body of mandible you hit the opposite side condyle is in jeopardy for fracture

-          What is the best way to look that the TMJ?(CT, MRI, pan,..)

-          When is it appropriate to do an I&D? (diffused cellulitis, large firm, flucuant localized,…)

-          What is the most common side effect of N0-O2 sedation? Nausea

-          The soft palatal is supplied by what structure? (pharyngeal n, nasopalatine a, greater palatine n & a, lesser palatine n & a)

-          What should you do after a pt has a larger swelling after initial antibiotic therapy? Take culture

-          During extraction, you get a 2mm, sinus exposure, what it the treatment of choice? (observe, graft, take xray & Rx antibiotics)

-          Pt is infected with MRSa, what antibiotic do you give? Vancomycin mosby pg 305 & 307 MOA- inhibits transglycosylase in cell wall synthesis. Narrown Spectrum.  Gram positive aerobe and aneroble.  Given IV.  Indications for staph.aureus including methicillin resistant staphylococci, Strept, Enterococci and Clostridium Difficule. ADVERSE EFFECTS- rental toxicity, ototoxicity, and “RED MAN SYNDROME”

-          Infection in the upper lip can go where? Cavernous sinus

-          Why is upper lip infection serious to treat? Cause facial veins lack valves

-          What is the earliest sign of cavernous sinous thrombosis? (Periorbital edema, decreased vision, ophthalmoplegia) Cavernous sinus thrombosis symptoms include; decrease or loss of vision, chemosis, exophthalmos (bulging eyes), headaches, and paralysis of the cranial nerves (III, IV, V, VI) which course through the cavernous sinus. This infection is life-threatening and requires immediate treatment, which usually includes antibiotics and sometimes surgical drainage.

-          What do you seen first with healing after SRP? Long junctional epithelium

-          Which drug is an antifolate analogue? Methotrexate (used for cancer tx)

-          Reduction of cusp is determined by what factor? (outline, convenience, resistance, retention)

-          -dose ratio of a drug is dependent on what factor? (clearance, distribution…idk)

 

 

 

 

KRISTIN:

 

Hope this help Ill post more as I remember Good luck

 

1.       Posture of patient affects what? VDO VDR protrusive, CR, etc

 

2.        HSV 1 what % US population? IDK I put 30% look it up Per Wikipedia, In the US, 57.7% of the population is infected with HSV-1[68] and 16.2% are infected with HSV-2. Among those HSV-2 seropositive, only 18.9% were aware that they were infected.[69] Worldwide rates of HSV infection are between 65% and 90 determined by the presence of antibodies against either viral species.[67]

 

3.       Patient feels helpless in chair how can you help? Let them raise hand when needed

 

4.       What is seen in diabetics? Murcomycosis Aka Zygomycosis. Found in insulin dependet diabetics w/ ketoacidosis, transplant recipients, and chemotherapeutic patients.  Murcomycosis is opportunistic fungal infection and found in bread molds, and decaying fruits or vegetables.  Symptoms nasal obstruction, facial swelling and visual disturbances.  If untreated leaves black and necrotic tissue in palate in maxilla.  Histo: nonseptate hyphae branching at right angles. Tx is surgical debridement and systemic administration of Amphotericin B

 

5.       Tooth erupts through tissue? ¾ roots complete

 

6.       Patient had flu shot never had a dental shot but scared as shit cause of flu shot what is this? Generalization I think KAPLAN HANDOUT_ GENERALIZATION- a different but similar stimulus brings on a result.  i.E. White uniform, handpiece/prophy. EXTINCTION- loss of a response after several times with no reinforcement. Extinction of a behavior or dental fear

 

7.       Patient kept having failure with veneer 3 times why is this? I put something with anterior guidance off why he kept fracturing them IDK Dental Board Busters pg 327. Contraindications of Porcelain Veneer- severe imbrication of teeth, traumatic occlusal contacts, unfavorable morphology, insufficient tooth structure and enamel. A patient with high caries index, short ccl crown, and minimal horizontal overalp.

 

8.       Porcelain veneer greater than composite veneer except I put 2 appointments more time to do

 

9.       Implant and something about fixing with angulations? I chose something with use angled abutment can’t remember

 

10.   How do you know if its perio over endo? Pain on lateral percussion……

 

11.   Maxillary permanent incisor had hypoplasia when did it occur? Utero, 1 month to 6 month…etc know calcification  Mosby pg 176

 

                 ****CALCIFICATION TIMES PERMANENT TEETH                                                                                                              

Time

Teeth That Begin to Calcify Permanent

 Birth

First molars

6 months

Anterior teeth EXCEPT Max. LI

12 months

Max.LI

18 months

First PM

24 months

Second PM

30 months

2nd Molars

 

TOOTH

MAXILLA

MANDIBLE

1M

BIRTH

BIRTH

CI

3-4 MONTHS

3-4 MONTHS

LI

10-12 MONTHS

3-4 MONTHS

C

4-5 MONTHS

4-5 MONTHS

1PM

1.5 YEARS

1.75 YEARS

2PM

2 YEARS

2.25 YEARS

2M

2.5 YEARS

2.75 YEARS

 


CALCIFICATION OF PRIMARY TEETH

TOOTH

CALCIFICATION

CI (A)

14 WEEKS IN UTERO

1M (D)

15 WEEKS IN UTERO

LI (B)

16 WEEKS IN UTERO

CANINE (C)

17 WEEKS IN UTERO

2M (E)

18 WEEKS IN UTERO

 

TOOTH ERUPTION

MAXILLA

MANDIBULAR

CI

10 MO.

8 MO.

LI

11 MO.

13 MO.

CANINE

19 MO.

20 MO.

1PM

16 MO.

16 MO.

2PM

29 MO.

27 MO.

 

NOTE: CALCIFICATION : ADBCE AND ALL PRIMARY TEEETH BEGIN TO CALCIFY IN UTERO AND ERUPTION: ABDCE WITH TEETH BCD ERUPTING EARLIER IN MAXILLA

 

12.   Post insertion denture case man had generalize pain on ridge? Not enough interoccusal space the VDO is too great

13.   Anxiety patient what drug can you give the night before to help sleep? Diazepam, clonazpam, ambien…etc  AMBIEN (ZOLPIDEM and ZALEPHON) is sedative hypnotic drug that works similar to benozodiazepines but on the BZ1 receptor type.  It is a long-acting drug and is taken orally.  It is used to treat insomnia and difficulty falling asleep. Benzo drugs enhance the effect of GABA at receptors on CL channels.

14.   Angles Class 1 occlusion what line up with what? Mesiobuccal cusp of 1st max in the buccal groove of 1st mand……need more question like this

15.   Child afraid of dentist and has never been who fault is it? I put parent a subjective condition

16.   Patient asks about Mask gloves and eyewear what’s the least response to say? Something reckless but you will know once you see it.

17.   Most dental treatment for the Aged is funded by who? I put out of pocket considering 60-70% of U.S. don’t have dental insurance…Medicare does not cover dental only Medicaid  Mosby pg 221 Medicare- insurance for elderly and disabled Americans. DOES NOT cover dental care except when dental services are directly related to the treatment of medical condition. i.e. extraction of teeth prior to radiation therapy for cancer.  MEDICAD- mandates that states provide early periodic screening diagnostic treatment for children.  It has optional adult coverage, but varies by state for benefits.  CHIP- is called Children Health’s insurance program and it provides basic preventive, diagnostic, and restorative services but not as broad as medicad

18.   Radiation to the operator is mostly from? I chose scatter for patient body IDK choices where stupid

19.   With removal of internal oblique ridge what could happen? I chose possible damage to lingual nerve

20.   The Dr. lavell stuff far as patient management is common sense but the cohort retrospective cross sectional…etc is very tricky and worded very crazy just try to know it cold I thought I did but clearly.. MOSBY PG 310-311

21.   Pregnant woman with hypertension? Preeclampsia

22.   ASA II

23.   Head and neck radiation what caries do you see most? I chose cervical VIA INTERNET Radiation or cervical caries are usually related to xerostomia and/or chemical changes in saliva. Radiation therapy to the head and neck region usually exceeding 4000 rads.is the most common cause

24.   Lady had white cloudy whatever on buccal mucosa disappear when stretched? Lukoedema  White lesion that is bilateral opacification in the buccal mucosa.  It has a whitish grey appearance that returns to normal when skin is stretched. It is seen in dark pigmented skin people and those who smoke

25.   Patient listens to favorite music on headphones during treatment what is this? Distraction, desensitization…etc Mosby pg 228. Treatment of managing anxiety. Distraction-giving the patient a competitive attentional focus(listen to music, watch tv), desensitization- exposing a patient to items from a collaborative hierarchy of slowly increasing anxiety provoking stimuli)related to target fear) while using relaxation skills. Others concepts to relieve anxiety are 1.)diaphragmatic breathing 2.) muscle relaxation 3.) Guided imagery- patient uses diaphragmatic breathing skills while imagining a pleasant scene of their choice 4.) hypnosis 5.) behavioral rehearsals- pt. has opportunity to practice coping strategy using diaphragmatic breathing while experiencing a simulated procedure or part of a procedure.6.) flooding- intense or prolonged exposure to a feared stimulus while using coping skills 7.) biofeedback- teaching one to have control over his/her physiological arousal through the use of auditory/visual monitoring. 8.) cognitive coping (reframing) assisting pts. With changing their thinking 9.) use praise- asking patients to practice coping skills at home and when in office.

26.   Non-working questions like 8 of them just saying thing…Lingual inclines of buccal on mandible and inclines on lingual cusp of maxillary

27.   Modified ridge lap pontic should be passive very little contact to ridge Mosby pg 332 -333. A pontic design is in 2 categories 1.) Mucosal pontics- ridge lap, modified ridge lap, ovate, conical or bullet shape.  All of these should be concave and passively contact rige. 2.) Nonmucosal pontics –sanitary (hygienic) and modified sanitary hygienic. These are generally in nonesthetic areas.  A saddle pontic design covers the ridge labiolingually forming a  concaeve area that is uncleansed and for that reason is not used.

28.   Bleeding time measure what? Platelet clotting, intrinsic factors, extrinsic factors, common pathway

·         Bleeding  time=The time it takes for bleeding to stop (as thus the time it takes for a platelet plug to form) is measure

 

29.   NSAID affect what?

NSAIDS affect the prostaglandin production,

30.   Aspirin and asthma is a no go!!! Can’t remember the? That’s the ans

31.   Free gingival graft what can you eff up? I chose greater palatine neural bundle….most of the grafts come from hard palate

FGG is the removal a section of attached gingival from another area of the mouth usu. hard palate or edentulous area to the recipient site. FGG is used to increase the zone of attached gingival and gaining root coverage. 

 Success  is when graft in immobile at recipient site.

FGG is dependent the blood supply of it recipient site.

FGG failure mostly due to disruption of the vascular blood supply before engraftment.  Infection is second most common reason of failure.

FGG indications:
prevent further recession and successfully increase width of attached gingival.

Cover non pathologic dehiscences and fenestrations

Performed with frenectomy

Cover a root  surface

To widen attached gingival after recession

32.   Acute asthma patient his bet blocker is not working what else can give? Epi I think

33.   Non odontogenic pain how can you tell? Give a block if patient is still in pain it’s not the tooth

34.   What injection will most likely cause hematoma? PSA

35.   The elderly suffer from depression

36.   ANB is a -6 what is it? I chose Class3 most of the time if ANB is negative its class 3

37.   After insurance pay their part the remaining that the patient pays is the? IDK deductible seem like the only one that made sense

38.   Salivation with denture effects retention more is better

39.   What is main purpose of splinting teeth? Patient comfort

40.   Someone had a delayed hypersensitivity reaction what can you give? I chose Diphenhydramine…there where two questions like this other ans was antihistamine same thing

41.   What’s a contraindication in doing a RCT? vertical root fracture

42.   Main factors when doing a posterior composite? I think I chose type of resin and depth of prep idk can’t remember

43.   What is seen in DI? Kind of tricky had two possible ans but I went with obliterated pulp think the other choice had something to do with osteogenes and blue sclera

44.   What will happen if you get to happy and add to much monomer with acrylic? increase shrinkage

45.   Reason for mucocele on lip?  Obstruction, minor salivary gland by mucus plug

46.   Which patient is most likely to have paint in joint? Sickle cell, hypertension...Etc RA was not a choice

47.   Which condition is most likely to have TMJ anklyosis as well? Rheumatoid arthritis

48.   Purpose of Plaque Index? Patient motivation seem like the best choice out of the bunch idk

49.   Why do you use irrigation when placing implant? Keep from overheating

50.   5 yr old child is acting out how do you stop it? Whip dat ass, naw I think Voice control for the boards

51.   Autistic child-likes affection, needy, repeat things over and over

52.   Patient had very old large Mod amalgam with margin intact but pain when trying to bite a piece of French bread and cold sensitivity?

53.   Patient had oral Candida all are possible questions to ask except? I chose something about him eating bread with no preservatives….

54.   30 plus yr old patient had deep fissures and pits what do you do? Observe

55.   Had a question of yield strength and ultimate strength don’t remember but know them

56.   Where can you most likely see a nutrient canal on an x-ray? Ans choices where all teeth except the molars IDK

57.   All the following are true of a schedule 2 drug except?

58.   Best way to treat retained roots in over denture? I chose something with microbial (keep them clean)

59.   Polyether material is very stiff

60.   Dentist biggest concern with a denture patient or something like this? I chose making sure to get the realistic factors across to the patient about the denture.

61.   Trauma to child mandible what is most common? Mandible asymmetry, hyperplasia…etc.

62.   Two question about mouth breathers and whatever with mandible plane angle

63.   Question about transferring a wheelchair person for wheelchair to the chair? Sliding board, ask patient, don’t use patient belt, etc idk I chose the last one WTF you need a belt for O_o

64.   Dens in dente mainly seen in which tooth? max lateral

65.   Most likely IAN parathesis at which fracture in mandible? I chose angle/body?

66.   Dentist is doing MOD on max molar when is it ok to cross the oblique ridge? When it’s less than 1.5mm left of tooth structure

67.   No calculation for me so glad cause all I know how to count is that money Pheeeewww

68.   Review Perio and know what type of organism are seen with each type including your endo ones

69.   09-10 remember is a good review not going to be the same but it’s a few I would def look at it

70.   Know INR

71.   Had a lot of RW post so this just and add on to that

72.   If you don’t know the ans go with C…..shrug seem to be the ans choice of the day

73.   Too much vertical angulation in xray....distortion, overlap,magnification, and sharpness ( the put distortion)

74.   What is reverse architect in peridontitis?

75.   Showed a pic of some 15 yr old boy mouth had excessive gingival inflammation all of a sudden for 5mos and not on meds also bruise easily? I said he had acute leukemia but idk

76.   Childs heartbeat look at chart in mosby pg 183 I put 110 but the child was 4yo and that was the best choice everything else was below 100

77.   Somebody is freakin out they feel doomed!!!!!! Forgot other answer chioce but I put panic attack.

 

 

Day 1:

Good Luck Hope this helps, this is just from the first day. 

 

Pephigoid, know calcifacations times, what effects xerostomia causes, Papillon le fever, definition of Papilloma,  

 

Regen of periosteum needs- sharpeys fibers, cementum, alveolar bone, periodontal ligament

Denture pt with opposing teeth? Mx bone resorption, post tuberosity droops, Mx anterior resorption (Combination syndrome)

 What cause angular chelitis-VDO loss

159. Palmar plantar keratosis- Lefevre Papillone

 #8 lighter than the rest of teeth what do you do? Bleach other teeth, crown

Pregnant woman in dental chair- lay on left side to prevent from laying on vena 

   Pt on antidepressant what is your greatest concern? Epinephrine or time in chair

These patients are usually taking MAOI which may potentiate the effect of the Epi by inhibiting the re-uptake.

Gardner syndrome- osteomas, polyps that turn into adenocarcinomas, supernumary teeth

 

H2 histamine receptor is for gastric acid 

INR determines PT measure warfarin dose, liver damage, vit k status

T test vs chi square (The t-test assesses whether the means of two groups are statistically different from each other)

Most:  3rd molaràmand 2 bicuspidàmax lateral (least)

Fentanyl is an opioid reversed by naloxone and flumazenil reverses benzodiazepine

Purpose of plaque index? Show the patient

Fluoride- how much do we use in community water 0.7-1.2 ppm

Main reason for redoing anterior composite-discolored

 

  Reason for mucocele on lip?  Obstruction, minor salivary gland by mucus plug, trauma

The mucocele constitutes the most common nodular swelling of the lower lip. These swellings are asymptomatic, soft, fluctuant, bluish-gray, and usually less than 1 cm in diameter. Enlargement coincident with meals may be an occasional finding. The most common location is the lower lip midway between the midline and commissure, but other locations include the buccal mucosa, palate, floor of the mouth, and ventral tongue. Children and young adults are most frequently affected. Trauma is the etiologic agent.

156.                        Why is 3 degree burn vs 1 degree burn less painful

First-degree (superficial) burns
First-degree burns affect only the epidermis, or outer layer of skin. The burn site is red, painful, dry, and with no blisters. Mild sunburn is an example. Long-term tissue damage is rare and usually consists of an increase or decrease in the skin color.

Second-degree (partial thickness) burns
Second-degree burns involve the epidermis and part of the dermis layer of skin. The burn site appears red, blistered, and may be swollen and painful.

Third-degree (full thickness) burns
Third-degree burns destroy the epidermis and dermis. Third-degree burns may also damage the underlying bones, muscles, and tendons. The burn site appears white or charred. There is no sensation in the area since the nerve endings are destroyed.

 

    Erosion- bullemia

Patients with bulimia may present with severe erosion of the lingual and occlusal surfaces of the teeth Severe erosion can cause increased tooth sensitivity to touch and to cold temperature. Dental caries may be more prevalent in these patients. The amount of saliva produced may be decreased. Patients often report dry mouth. Those with poor oral hygiene have increased periodontal disease. The parotid gland may become enlarged, and patients with anorexia nervosa may have decreased salivary flow, dry mouth, atrophic mucosa, and an enlarged parotid gland.

Aspirin: associated with Asthma

  

Contraindication for implant-myocardial infarct, smoking, bone loss

 

Periostat n doxycycline inhibits what?  collagenase

 

Ectodermal dysplasia

Hereditary ectodermal dysplasia 

1. An X-linked recessive condition that results in partial or complete anodontia.

2. Patients also have hypoplasia of other ectodermal structures, including hair, sweat glands, and nails.

 

69.   How long do you take patient off of Coumadin before surgery? 2-3 days

The INR is used to gauge the anticoagulant action of warfarin. Most physicians will allow the INR to drop to about 2.0 during the perioperative period, which usually allows sufficient coagulation for safe surgery. Patients should stop taking warfarin 2 or 3 days before the planned surgery. On the morning of surgery, the INR value should be checked; if it is between 2 and 3 INR, routine oral surgery can be performed. If the PT is still greater than 3 INR, surgery should be delayed until the PT approaches 3 INR. Surgical wounds should be dressed with thrombogenic substances, and the patient should be given instruction in promoting clot retention. Warfarin therapy can be resumed the day of surgery

(Hupp, James R.. Contemporary Oral and Maxillofacial Surgery, 5th Edition. Mosby, 032008. 1.3.6.2). 

Warfarin and Coumadin are oral anticoagulants that inhibit the biosynthesis of the vitamin K–dependent coagulation proteins (factors VII, IX, and X and prothrombin). These drugs are bound to albumin, metabolized by hydroxylation by the liver, and excreted in the urine. The PT is used to monitor warfarin therapy because it measures three of the vitamin K–dependent coagulation proteins: factors VII and X, and prothrombin. The PT is particularly sensitive to factor VII deficiency. Therapeutic anticoagulation with warfarin takes 4 to 5 days.1

Level of anticoagulation and need for altering dosage to avoid excessive bleeding

PTR (1.5 to 2.0) or INR (2.0 to 3.0): Dosage does not need to be altered

PTR (2.0 to 2.5) or INR (2.5 to 3.5): Dosage may be altered

PTR (2.5 or >) or INR (3.5 or >): Delay invasive procedure until dosage decreased

Decision is made to alter dosage of anticoagulation medication

Physician will reduce patient's dosage

Affect of reduced dosage takes 3 to 5 days

Dental appointment needs to be scheduled within 2 days once desired reduction in PTR or INR has been confirmed

 

67.   Null hypothesis

the null hypothesis, which is the hypothesis that there is no real (true) difference between means or proportions of the groups being compared or that there is no real association between two continuous variables

65.   Denture for 19years- relieve  pain denture and have white spot what do you do

Relieve the denture in the area of the lesion and reevaluate  in 1 week.

 

47.   Incidence 100/1000

Incidence: indicates the number of new cases that will occur within a population over a period of time (e.g., the incidence of people dying of oral cancer is 10% per year in men aged 55 to 59 in our community).

 

37.   Nevoid BC

Nevoid basal cell carcinoma syndrome (Gorlin syndrome) is an autosomal dominant inherited condition that exhibits high penetrance and variable expressivity. The syndrome is caused by mutations in patched (PTCH), a tumor suppressor gene that has been mapped to chromosome 9q22.3-q31. Approximately 35% to 50% of affected patients represent new mutations. The chief components are multiple basal cell carcinomas of the skin, odontogenic keratocysts, intracranial calcification, and rib and vertebral anomalies. Many other anomalies have been reported in these patients and probably also represent

Periapical cemento-osseous dysplasia

1. A reactive process of unknown cause that requires no treatment.

2. Clinical features

a. Commonly seen at the apices of one or more mandibular anterior teeth.

b. No symptoms; teeth vital.

c. Most frequently seen in middle-aged women.

d. Starts as circumscribed lucency, which gradually becomes opaque.

An exuberant form that may involve the entire jaw is known as florid osseous dysplasia

 

Manic depressive not taking medicine what will happen? Mood swings

 

17.   What turns porcelain green? Copper or silver

 

Porcelain that is baked onto a high –fusing gold alloy may exhibit a green discoloration due most likely to contamination of the metal by COPPER traces. (Dental Decks)

Examples of metallic oxides and their respective color contributions to porcelain include iron or nickel oxide (brown), copper oxide (green), titanium oxide (yellowish brown), manganese oxide (lavender), and cobalt oxide (blue). Opacity may be achieved by the addition of cerium oxide, zirconium oxide, titanium oxide, or tin oxide

13.   LED curing vs regular curing? Why is LED curing beter? Range, last longer

Most recently developed are the LED curing units. These units have a number of advantages compared to other curing units, including a wavelength spectrum emission that is closely matched to camphorquinone. In addition, these units are more energy efficient, allowing them to be battery operated. The diodes have a life span that is approximately 1,000 times longer than the typical halogen bulb. While the earlier versions of LED curing units provided inadequate irradiance, the newer generation has overcome this deficiency. About the only disadvantage to these units is their narrow wavelength spectrum, limiting their usefulness in curing any materials that do not use camphorquinone as the photoinitiator.

The practical consequence is that curing depth is limited to 2 to 3 mm unless excessively long exposure times are used, regardless of lamp intensity. 

Implants: 3mm from another implant

               5: mm from mandibuar canal

               ? from adjacent tooth

 

SW:

1.       Which test is MOST valuable in an tooth that needs testing that has an open apex…young tooth = cold test (EPT does NOT work)

2.       Internal bleaching will MOST likely cause? Extracanal cervical resorption, replacement resorption, external resorption (idk? Was not aware of what “cervical” resorption was especially when given extracanal cervical resorption AND external  resorption in answer choices???)

3.       Apexification-done when tooth is NOT vital (aka need RCT) and you need to close the apex so you can get a seal for the gutta percha; Apexiogenesis-done when tooth IS VITAL and RCT is NOT necessary no matter what has happened (but you have to know the situations in which a tooth would be vital or nonvital)

4.       When tooth is traumatically intruded… LET IT ERUPT!

5.       Difference between reversible and irreversible and necrotic SYMPTOMS… how long pain lingers to COLD test etc.

6.       Sensitivity to percussion and biting you know you have acute apical periodontitis

7.       MOST likely cause of pulpal inflammation= DECAY/BACTERIA/CARIES etc.

8.       Pain from which one, mandibular premolar or mandibular molar, refers to the ear? Idk? I have a hard time choosing between the two

9.       SLOB rule question, Buccal root shot from M, now shoot from D and its oppositie the lingual root shot from mesial blah blah blah (SAME LINGUAL OPPOSITE BUCCAL)

10.   Vertical root fracture= EXTRACT

11.   Disadvantage of NaOCl: toxic to tissues (does NOT remove smear layer btw)

12.   Difference b/w self etch and total etch: self etch does NOT remove smear layer

13.   Bacteria responsible for pulpal infection: not specifically which one but answer choice was one or more than one bacteria? Idk

14.   Ledge… bypass the ledge

15.   Tooth most likely to have 2 canals: max 1st pm

16.   Where canals are in mand molar most likely: 3= 2 M and 1D

17.   Which access preparation canals look like a C shape? I forget mand molar?

18.   What facilitates RCT… NOT calcified canals

19.   pH 5.5 critical

20.   treatment sequencing Mosby p. 38

21.   problem with amalgam lies in environmental disposal

22.   difference b/w resistance and retention forms

23.   knowing that liners and bases and recurrent decay can all appear radiolucent

24.   on a pan 2 bones that can appear below mandible= hyoids

25.   place calcium hydroxide then resin glass ionomer base then restoration

26.   admix= spherical and boxy particles

27.   breakdown of a composite margin most likely due to: shrinkage during curing, expansion, or overfinished margins? Idk

28.   large posterior decay: large composite is NOT ideal

29.   COTE = gold< amal< unfilled composite

30.   Alveolar osteitis= dry socket

31.   Bleeding after ext 3 hours later à remove clot to locate location of problem

32.   Pt has 3rd molars… what do you do: tell the patient 3rd molars cause crowding and need to come out, do not inform patient about them, 3rd molars are associated with cysts and various pathologies and need to be extracted, or do not do anything about them

33.   Pt has cirrhosis of liver: what is his liver mostly composed of? Hepatocytes, fibroblasts, hematopoetic cells… idk

34.   Pan had a 3rd molar that was basically straight up and down maybe tilted to distal by 2 mm but erupted fully… it was NOT distoangular impaction (know what these look like on an xray-distoangular, mesioangular impaction)

35.   Day after ext pt comes back with fever and sick feeling… give him different antibiotic (bc he has AA bacterial infection which is associated with ext is what I thought) OR refer him to proper specialist… idk

36.   Adult pt has crossbite needs fixin: listed 3 ortho appliances-even quad helix (tricky), or surgery- adult so I chose surgery

37.   Distraction osteogeneis good for LARGE movements

38.   ANB 6… Class 2 dental or Class 2 skeletal malformation?

39.   Trigeminal neuralgia is NOT associated with a spontaneous dull ache

40.   Pt reports 3rd molars have been ext years ago… pan reveals small round RO in area of #17: osteitis

41.   Pt has cirrhosis of liver: what is best: lidocaine, mepivicaine, bupivicaine, articaine

42.   Infected tissue… LA will be in ionized form

43.   Prilocaine- methemoglobinemia

44.   Swelling in front of SCM:

45.   Cleft lip/palate 1:700

46.   First ‘dental formation’: (weird!) 6 weeks, 16 weeks, 32 weeks idk (teeth, palate, what idk what you’re talking about…)

47.   Syphilis lesion that looks most like herpes? Idk

48.   Pemphigoid against “basement membrane” (pemphigus against something else)

49.   PCOD ant aa female ANT MANDIBLE

50.   Xray teeth with no pulps: DI pg 124 mosby

51.   Target made of tungsten

52.   Mosby 141 post wall of zygomatic process on xray

53.   Man w/ ill fitting partials…indurated ulcerated lesion lat border of tongue: SCC or trauma from partial? SCC and biopsy

54.   Untreated decay mostly in AA… oral cancer mostly in AA males mosby 207

55.   Pg 163, 164, 165 know like back of your hand… quad helix corrects crossbites, use these all in GROWING ppl, surgery for adult…when to use LLA or band loop PLEASE KNOW THIS! LLA-bilateral loss, #19 and 30 NOT erupted yet= need distal shoe… if they are erupted need BL

56.   Chi square= categories, ttest=averages or means

57.   Modeling pg 225 mosbys

58.   Probing + recession= clinical attachment level

59.   Do NOT attempt perio surgery until you have tried and failed at initial SRP therapy

60.   45-90 angle on SRP instruments

61.   Most cost effective: stress oral hygiene home care

62.   Pg 272 intrinsic activity and maximal effect and efficacy and receptors and affinity just know it all and how its all interrelated

63.   Beta blockers end in –olol, anti GERD drugs –prazoles (omniprazole=tagament/prevacid etc)

64.   Pharm: carbamazipene, atropine, mechanism of tricyclic antidepressants

65.   If you change vertical dimension occlusion during fabricating a complete denture what do you have to REDO? CR or facebow… CR!

66.   How to help pt who gags with their denture? Tell em to put the denture in for as long as they can, put a spoon and hold it for as long as they can tolerate it-do this over and over-YES

67.   Implants 3mm apart

68.   323-324 mosby denture phonetics… just know why you make all the sounds ALL of them, esp f,v, ph (do it yourself and you’ll remember)

69.   Kennedy classifications must know cold

70.   Value is lightness or darkness

71.   Primary tooth with most buccal and lingual convergence…idk

72.   Hemoglobin type in sickle cell disease: A, C, F, or S? idk S

73.   Pan with laterals missing in photograph but present on pan just impacted, 9 years old, canines impacted… (#1 dental age conincides with his chronological age-NO #2 dental anomaly in this kid occurs more in permanent than primary dentition, this occurs in proliferation stage-TRUE OR FALSE idk)

74.   Pan with 2, 14, 15 w/ 50% bone loss and bombed out 14 and 2 and unilateral edentulous area on mand… tx plan? A.crown lengthening and crowns B.orthodontic tx on maxillary C.max class 2 partial D. MAND CLASS 2 PARTIAL

75.   Radiolucency on pm with MO amalgam could be all EXCEPT: recurrent decay, liner, base, something off the wall…

76.   “your fees are high!” your response should be? “it seems you are concerned if getting your dental care is worth it?” OR “ my fees are the same as all the other dentists” or “I have to pay bills” blah blah

77.   Modeling… use sister or other kid to show uncooperative kid what to do

78.   Reason for everything: PLAQUE


JAK:

AOT, Ameloblastoma, LPC etc—know cold—ie location
Implant measurements
Soldering
Flaps

GTR

Systemic fungal antibiotic prophylaxis

Calculate lidocaine for a child

Child head shape

Fontanelles

Pierre Robin

Non-chelating—naocl

Crevicular fluid stuff—nomal—diabetic etc

Insurance stuff

Ethics

Ceramics

Ulcer/herpes—know cold its tricky

Xerostomia—causes

Turodontism/pulp stones?—occlusally, apically, both

Most commons—a lot ie impacted

Abnormal eruption

Childhood caries

Cleft palate

Xray/max per year percentage between d

Kvp and ma

Difference between round and rectangular cone beams

 Bisphosphinates

Distance (doubled)

Max xray per year/week

Know how to read ceph

 

 

AP:  Greetings Fellow Doc’s….. First and foremost, thank you to everyone who posted there remembered questions thus far.. It helped me immensely!!!!!!!!!!!!!!!!!!!!!!!!!!!!!!!! I want to let you know that the exam is very fair and if you have studied you will do well ….. I would definitely review the classes remember questions as well as the 09-10!!!! If you know all of them, you will have a nice bunch of “gimmie’s” before it starts to get real….. LOL, There are some tricky questions though but put your thinking caps on….

 

Pics I had to ID were: All From RW test… I had his entire exam…..  and KG’s so I will only add things that I can fill in the blanks to or that were not present.

THANKS R2!!!! ;-) and KG!!!!!!!!!:-)

 

-Regional odontodysplasia

A dental abnormality of unknown cause ; genetics trauma, nutrition and infection have been suggested.Quadrant of teeth exhibit short roots, open apices and enlarged pulp chambers.

The radiographic appearance of these teeth has suggested the term ghost teeth.

-Zygomatic process of the Maxilla= pg 141 mosby

PAN point to a radiopacity anterior the poster border of the max sinus

-What symptom will a person have with trisomy 21 have? Small midface ( deficient midface)

 

-Why would you reduce the opposing dentition before doing a fixed bridge

                Establish adequate occlusion

                Remove interferenece before to eliminate reducing them later

                ……. Cant really remember the others at the moment.

-What does primary tooth discolor after trauma?(diffusion of blood in dentin tubules, fibrosis of pulp, internal resorption, 1more choice)

 

-When removing the internal oblique ridge, what is the risk associated?

Damage to the lingual nerve, other choices…… That’s the answer I went. I agreed with Krisitin!!

 

-Bleeding time measure what? Platelet clotting, intrinsic factors, extrinsic factors, common pathway

-after extraction patient is experiencing hemorrhage 3days later. Why?

                - vagal something….  Sorry that’s all I can remember.  That is the only answer that seemed to be the most correct to me.

-Patient has a3mm palate constriction resulting in a unilateral posterior crossbite. What does he to to maximize is occlusion?

-          Functional shift to the side of the crossbite

-          Functional shift to the opposite. ( I chose this answer)

-          Anterior open bite

-          One more selection

-What is the plaster record used  for the articulator after mounting the maxillary cast???

                Centric relation

                Establish the vertical dimision of occlusion

                To not mess up the transfer of  the facebow record.

What causes green and orange staining on the anterior teeth.

                Diet

                Ingestion of metallic substances..

                3 other choices.. I chose

 

What  can you give a medically compromised child to help with plaque accumulation.

                Fluoride

                Chlorohexidine

 

What does not involve measuring variation:

       Range. , variance, standard error , mode

                (don’t really know what this is,,,, so that’s way I selected this. )

 

Had a lot of questions for patient management. Most I have not seen on other remembered Q’s.

 

-A test is supposed to test positive for 95 out of 100 cases that contain a particular disease. This test is considered.

                Sensitive, specific, reliable, valid…..

               

----Desensitization questions (5-7 questions)

----Distraction- Patient is listening to headphones while performing a dental procedure……

----OSHA---- like 5-7 questions- what falls under OSHA policies and procedures.

----Components of a scientific article: title, abstract, Introduction, methods results and discussion.

 

 

Had a lot of Removable questions……….. What the F**K!!!!! ( thanks daphne!!!!! NOT)

 

                These questions were worded extremely funny so I really can’t remember how to articulate them..

 

 

They asked about distal extension/ distal extension vs. tooth borne RPD’s- what is the different between the distal extension vs. the tooth born RPD.

 

Bilateral distal extension …….. Just know the effects of indirect retainers, rests and there actions.

 

KNOW Composites!!!! Questions were funny…… interpreting the question is 90% of the answer…..

Had a lot o perio related to scaling and root planing,periodontal maintenance, endo perio lesions, perio-endo lesions etc……

 

I PLAN TO ADD MORE AS I REMEMBER. FOR THOSE TAKING THERE EXAM TOMORROW I WISH YOU THE BEST.. SORRY I COULD NOT REMEMBER ALL THE ANSWER CHOICES AND EVEN SOME OF THE ANSWERS I SELECTED. LORD KNOWS I TRIED, BUT WHATEVER I RECOLLECT I WILL BE SURE TO ADD ASAP!!!

 

PEACE CREST KIDS!!!   AS MY DAD CALLS US ;-)

 

PC:

Some repeats from class and some from 9-10…not a lot. TRICKY QUESTIONS….

Know wht mercury toxicity does?

PERIO PERIO PERIO….DIAGNOSIS,TX,GRAFTS,FLAPS,ANTIBIOTIC THERAPY…Bevel incision also,

Selective grinding for occlusion..Non working and Working

PCOD,GHOST IMAGE,CEMENTOBLASTOMA..ID IMAGE

Cohort,cross sectional,clinical trial

Veracity,bene,nonmal,autonomy

HIPPA,OSHA

Partial and bridges…know indication and how to adjust them

Noble metals..which one is  a noble metal?

Orthro appliances indications…1st molar and 2nd molar

Ph pka calculation…

Modeling,Desentization,Behavioral..know all  tht SHIT..

If I think of some more I will post it...

 

Pearls:

 

Ok my day 2: It was EXACTLY the same cases from the 09-10 which another classmate posted…so you have the cases…basically the exam writers were lazy …if they don’t change anything for the next couple weeks, everyone should pass, because it’s a rock…and day 2 is an EAGLE ! I will say that you still need to know the information because it seemed like it could be more than one answer for the majority of the questions, so I wouldn’t necessarily pick the same answers …pick what you think is best….basically if they’re not changing the cases, review ortho, operative, tufts pharm, and treatment planning, And don’t make the same mistake I made, I ran out of time and left the last four questions unanswered.

 

I honestly don’t think they’ll change it next week…but you never know….and as for January…who knows?? Best luck to all of you J

 

 

Day one:

Ok….sooo basically any question that I saw repeated was from the 09-10 and what everyone else has posted…so definitely look at those

 

I wouldn’t memorize all because some of the questions are asked differently…so just know the topic (I realized that they were just asking the same questions differently and maybe changing around the answer, because I would get to one and tell myself I just saw this a couple questions before)

 

Public health came from somewhere in mars….i don’t really know where to tell you to look because I read mosby, dental decks, and dental secrets and didn’t see the words they were using….I guess google/ wiki the types of insurance.. HMO/PPO/ etc …know more than just the basic definition

 

And just in general…know mosby’s public health like the back of your hand…I wouldn’t put a lot of effort into memorizing sterilization stuff…only question I had was the disinfectant definition.

 

Question about what would not be included in HIPAA …something about PHI and what is covered under the regulations ….(ummm ??? some type of “private health information”? lol Idk…maybe I’m slow )

 

A question about RL at the furcation of a primary molar?? Necrosis, trauma, normal anatomy, erupting premolar…

 

A dumb question about what type of mercury is used in dentistry….ethylmercury, methylmercury, normal mercury, ….who the f cares

 

A lot of really “nit-picky” questions on porcelain and veneers….and properties

 

Patient just has percussion sensitivity .. no other clinical/radiographic symptoms…what is it? Periapical abscess, periodontal abscess, APP, and I think CPP was also a choice

 

The sequence of extractions and why…but they only gave the max molars (so like 1st 2nd 3rd )….and none of the answers had anything to do with protecting mandibular sockets, etc…

 

Oral DX: was a lot of basic path …this was actually the easiest section in my opinion…PCOD twice, complex odontoma,  mucus retention cyst…only got like three questions on radiology about overlapping, etc….then a few identifications ….nutrient canal, pterygomaxillary fissure, mucus cyst in sinus…etc

 

Pharm: pretty straight forward…straight from tufts, exact questions actually…but it was a good number of questions where they would give you symptoms of like overdose or toxicity, and you had to pick the associating drug…or ask you what would you do first in certain situations …of course the pregnant lady ?s …bingo! (right hip up…duh)

 

Endo: simple stuff like lingering pain, what is it?....

 

Pedo/Ortho: mostly pedo…know calcification times, I had 2 questions, a whole lot of common sense behavior stuff…some weird question about movement during orthodontics…so just understand the changes that occur in the pdl/ bone etc

 

Operative, Prosth, and Perio were the ones that were really specific…like where in the oral cavity would you do/not do certain flaps and why, specific measurements for crown preparations (veneer, all ceramic....I don’t remember seeing anything on gold…wouldn’t waste my time memorizing that ish..just basics)….

-Operative…know composites cold…

 

most questions were specifics on like the results from water:powder ratio’s, overtrituration , what causes expansion of different materials…

 

one question about you do an MOD and the patient experiences sensitivity to cold or something…you take out the occlusal portion and redo it, that relieves the symptoms, so what was the sensitivity caused from? Answers were worded strange: something about gap causing bacteria to get into dentin tubules, can’t remember the rest

 

-couple simple implant questions….the one about 2 stage implant is mobile…I chose it failed so remove it…

 

Overall, it’s fair…the problem is they speak in 1800’s slang…so it’s hard trying to figure out what they’re asking… they use weird terminology…they won’t give you words or drugs you’ve seen…but their “made-up” words will have that word in it.

 

For example, they asked what drug is used to treat Herpes, Chickenpox, and CMV, answer choices were Amantadine, a fungal one, a retrovirus drug, and then paracyclovir…?? What? I suppose that’s acyclovir…so that’s what I chose.

 

Splash:

If you still have time, do those Kaplan q-bank questions. They were the truth.com

 

Eye-related accidents at the office- most often affect: Dentist, hygienist, dental assistant, custodial worker

 

Puncture wounds at the office come from: during the procedure, clean-up, re-capping the needle

 

Most common impacted tooth- mand 1st pm., max 2nd pm, max canine, mand canine

 

Access opening in maxillary pm, which wall is most likely to be perforated? Mesial, distal, palatal, buccal

 

Xray: identify u shaped radiopacity around max 1st molar: zygomatic process

 

Best xray to view zygomatic handle: Pan, Occlusal, Waters, one more (submento-vertex wasn’t a choice)

 

Mechanism of Action of Carvidopa- why is is good to use for Parkinsonism

 

All of the following cause gingival hyperplasia except: answer was Acyclovir

 

The role of microbial plaque is most obscure in which of the following diseases: gingivitis, periodontitis, desquamative gingivitis (what I chose) ,Necrotizing ulcerative gingivitis

 

Most of government funds for dental treatment come from: Medicare, Medicaid, HMO, grants

 

Immunofluorescence and IgG is used to diagnose: Pemphigoid, Erethema Multiforme….

 

How to treat ANUG

 

Porosity is most often seen in porcelain when..Firing temperature is too high

 

Stridor is diagnosed as bronchospasm, laryngospasm…

 

Patient in dental chair and starts expiratory wheezing…probably due to…asthma, copd,…

 

Most common dental emergency…syncope

 

Coronary artery disease in children linked to….obesity

 

Population of 1000. Last year 200 cases of disease. 1 year later…300. Whats the incidence? 100/1000, 400/1000…etc

 

Max. dose of lidocaine you can give to a 3 year old that weighs 16 kg? 10 kg, 54, 72, 115

 

24 month old presents to the dentist with 14 teeth with caries? Whats the best way to treat? Premedicate and local anesthesia, nitrous oxide and local anesthesia, general anesthesia, local anesthesia and restraint

 

Xray with description: 19 year old, scalloped radiolucency, teeth are vital? Traumatic bone cyst

 

Decreased alkaline phosphatase and premature exfoliation of primary teeth? Hypothroisim, hypoparathyroism, hypophospatasia

 

What is associated with osteomas? Cleidocranial dysplasia, gardeners…

 

What malignant salivary glad tumor has best long term prognosis? Polymorphous low grade, mixed malignant, adenocystic….

 

How to treat a ranula? Excise the top of the lesion, excise the submandibular gland, aspirate…

 

Mucous retention cyst most commonly located…lower lip, floor of mouth

 

Oral lesions of tuberculosis..

 

Lesions in mucormycosis resemble…ulcers, swelling of the tonsils…

 

Patient has endo/perio lesion..how do you treat? Endo first, possible perio; perio treatment only, perio treatment then endo

 

Couple questions on prevalence, incidene, etc

 

Freud and somebody else say that Anxiety is…. (definition of anxiety)  ie. Normal behavior, needs to be suppressed for proper maturation…

 

Lil boy ( I think he was 7) has severed decay of primary mandibular 2nd molars…distal shoe, bilateral band and loop, lower lingual arch…

 

Piaget’s theory of….

 

What appliance causes maxillary advancement- high pull head gear, cervical pull, protraction head piece

 

First symptom of cavernous sinus thrombosis- loss of vision, swelling of the eye…

 

Highest population of oral cancer- black males

Purpose of acid etching

 

Porcelain is strongest under compressive forces

 

Antidote for benzodiazepines- flumazenil

 

Morphine affects what receptors: Mu, Delta…

 

Opiod overdose….

 

Patient has swelling of lower left eye lid and can not feel the skin on the left side of the face..what is fractured? Roof of the orbit, floor of the maxillary sinus, medial wall of the maxillary sinus..

 

Endo diagnosis…

 

Most important when treating the elderly- speak slowly and clearly, involve them in the decision making process,….

 

Alkylating anticancer drugs…MOA

 

Place a composite..its too light. What do u do? Replace, composite tint…

 

Patient has shrunken face appearance….what needs to be done? Increase VDO, Decrease interocclusal distance, etc

 

Purpose of the facebow

 

Disadvantage of Polyether

 

Cementation of PFM. What do you check first? Margins, contour, esthetics, Interproximal contact…

 

Best drug to give for HIV patient with oral candidiasis? Systemic fluconazole, Topical something

 

Nitrous Oxide is contraindicated in pregnant women. When is best time to administer? 1st tri, 2nd tri, 3rd tri, No tri

 

Most common symptom/ side affect of Nitrous Oxide- muscle relation, bronchospasm, bradycardia..VOmitting or nausea was not a choice

 

Know when PTT, PT is prolonged, shortened (inder what circumstances)

 

Antidote for Acetominophen

 

Aspirin is contraindicated with what? Benzodiazepines, Tricyclic antidepressants, 2 other choices

 

Patient has wear face on the mesial incline of the mesiolingual cusp of max. 2nd molar…where else can  u expect to find wear on the mandible…distal incline on distobuccal cusp, mesial incline on db cusp, mesial incline on mesiobuccal cusp, distal incline on the distobuccal cust…

 

Best way to detect a fracture tooth? Xray, visually, Have pt. bite down locally

 

If you want to reduce pocket depth and not

 

Titanium is best used for implants due to its….biocompatibility

 

Absolute temperature in bone before necrosis after 1-5 minutes? 51, 55, 43, 47

 

 

Patient gets some kind of infection after administering an IANB? What space is it most likely in…pterygoid, cant remember the others

 

Whitened a tooth, best time to place composite restoration so that you don’t mess up the bond or something like that? 1 hour after, 24 hours, 3 days, 1 week

 

Most common complication with restoring implants? Mobility of the implant, loosening of the screw…

 

Type of tissue found between bone and implant? Type 1 collagen, sharpeys fibers, fibronectin

 

People who truly have the disease- sensitivity

 

Know how to treat fear and anxiety

 

Need for caries- plaque, bacteria, host..

 

Patient is taking hydrochlorothiazide…what test do you need to take? PT, PTT, electrolyte…

 

Patient in the chair and feels ill…what do you do? Lay them in trendelenburg, administer oxygen…

 

Pregnant woman 8.5 preggers and feels light headed. What do you do? Turn her slightly to the left, lay her back even more…

 

Best way to prevent osteoradionecrosis,…pretreat with hyperbaric oxygen, extract hopeless teeth in field radiation, extract the teeth with no precaution

 

Treatment planning sequence…

 

Tetracycline stains what? Enamel, Pulp, Dentin, Cementum

 

Biologic Width- 1mm, 2mm, 3mm,4mm

 

Gingival margin on Tooth #29 is 1 mm below the cej, pocket depth is 3mm. whats the attachment loss?

 

Most important factor for retention of crown? Axial taper, retention grooves,

 

Fixed resin restoration is at least 30degrees, 180, 270

 

What best benefits an edentulous  patient….removable dentures, over dentures supported, fixed implant prosthesis

 

Patient asks you to change a date on insurance claim. That is…FRAUD

 

U tell patient they need to change their amalgam fillings to composite…what ethics principal are you violating? Veracity

 

Patient has right to self governance..Autonomy

 

Adverse effects of glucocorticoids

 

MOA or adverse effects of Sulfonyureas…cant remember what the question was

 

Prophylaxis of Angina- propranolol, digoxin….

 

Propanaolol MOA….blocks beta adrenergic receptors…

 

Patient has a denture. Complaining of burning sensation of lower lip…..compression of the mental nerve

 

Radiolucent lesion around the pericoronal tooth…..Dentigerous cyst

 

Giant cells are associated with…hypothyroidism, hyperparathyroidism….

 

Acantholysis…Pemphigoid, Pemphigus, EM,…

 

Why the hell do you bevel the max. occlusal rims? 4 long answer choices

 

Prevalence of cleft lip/palate

 

Cleft lip palate- most affected by age, genetics,…..

Pan- had to identify soft palate

 

Lidocaine vs epinephrine effects…what casuses what

 

Discoloration of a recently traumatized lower incisor is due to what? Chromogenic bacteria, blood in the dentin, internal resorption

 

Replacement resorption- know causes and characteristics of it

 

Maxillary advancement- lefort 1, distraction osteogenesis, bilateral sagittal split…

 

What muscle is involved in facorable fracture? Temporal, masseter, mentalis..

 

Emergence profile of an implant- 2-3 mm apical to CEJ

 

Bone graft with most predictable outcome? Auto, Allo, Xeno…

 

Palatal tori removal when fabricating a denture…know when and when not to do it

 

Patients ridge…undercuts on tuberosities, for denture fabrication. What do you do? Nothing, reshape one for favorable insertion, remove undercuts on both…

 

Why do you do sinus lift? Something about implant placement

 

Indications for 3rd molar extractions

 

Indications for use of calcium hydroxide

 

Sodium hypochlorite…all of the following are properties of…except; Ans: chelating agent

 

ANtiobiotic Prophylaxis: Mitral Valve Prolapse WITH regurgitation

Antibiotic prophylaxis: Prosthetic heart valves

 

Intercuspal distance is greater than 1/3. What type of restoration? Amalgam, onlay, inlay, crown

 

Reducing cusp is used for Retention form, resistance form, convenience

 

Carbide bur with many blades…increase efficiency and smoothness….all those different options

 

What type of bur gives smoothest surface: cross- cut, fissure bur, diamond…

When do you restore a tooth? When still in enamel, when it reaches the dej,

 

What will not get rid of interproximal plaque? Proxy brush, tooth pick, floss, toothbrush

 

Dentists most likely see what clinically in bulemics? Enamel erosion….

 

Gastric acid most likely causes what? Abrasion, abfraction, erosion, attrition

 

Specifics about Pin placement- 1.5 mm axial to dej, all kinds of choices. (Refer to mosbys)

 

Advantage of Glass Ionomer….release fluoride

 

Angulation of the blade for SRP

 

Dentic sensitivity…hydrodynamic theory

 

Most common place for caries? Apical to contact, coronal to contact, fissures of  occlusal surfaces…

 

Worst prognosis? Perforation at apex, 3mm apical to apex, 3 mm coronal to gingival margin, 3 mm apical to gingival margin

 

Recently intruded permanent tooth. First step in treatment? Splint, rct, reposition, nothing

 

7year old. Receives .2ppm of fluoride? How much do you supplement? 1 mg,  .5, .75,. .25

 

Acidulated fluoride concentration? 1.23%, 5%....

 

When should patient sign informed consent? During treatment planning….

 

Material most likely to abrade natural dentition: gold, amalgam, composite, porcelain

 

Contraidications for implant placement: cardiovasculardisease, smoking, radiation therapy

 

Dentist agrees to providing serices to a certain group of people for certain fee. Funds run out. You can now, charge your regular fee, work out agreement with patient, honor fees…

 

What affects the posterior palatal seal? Vibrating line, hamular notches, (it was a list of things for each answer choice)

 

 

Day2 :

 

I had every case from 09-10.  Every question was asked, I’m just going to add on the extra questions I had to each case or add the reworded version.

Case 1

Long face, female, does not want to have those spaces between teeth – needs to close them (she does not have Lateral Incisors) Good oral hygiene.

1). What treatment for flare out of Central Incisors? a). With removable dentures b). Braces  c). Face-gares

 

 Case 2

AA man in his 40s. not currently taking any medications, or has allergies to anything. Doesn’t receive regular medical or dental health.I forget the CC.  But under patient info it states: Patient didnt get regular treatment. Now that he has insurance he can afford to get regular treatment. He wants to get his teeth fixed because he wants to keep all teeth that are worth saving. Tells the dentist that he has some mild anxiety towards going to the clinic. Had an emergency rct on #18 2 months earlier that was completed successfully.

AA man 46 yrs afraid of dentist, #3 and #6 crown decayed completely – lost.

1). Low blood pressure, low pulse  - Syncope when have anesthesia

 -The vitals of the patient show very low blood pressure and pulse. what is most likely to happen?  A) cardiovascular collapse, B) Thyroid storm C) Syncope D)some else that wasn’t right. 

2). Fix #8,#9 esthetic edge/yellow stain. What treatment to #8, #9? (little bit cheeped off enamel of #8) a). polished to smooth b). composite c). bleach

- Fix #8,9 the patient doesn’t like how the two front teeth aren’t on the same level. What would be the best treatment? A). place a composite restoration on both teeth to put them on the same level.  B). Place PFM crowns on both teeth. C). Smoothen out the distal portion of the incisal edge of number 8. D). Place a porcelain veener on the both 8 and 9.  The thing about this question is that the distal edge of number 8 does look off angle, but the problem is that the chip is on the mesial of 8 not the distal. (it’s a really small chip too).  #8 does look a little off colored on the distal facial side but they didn’t ask about what to do about it.

3). What treatment for #3, #6 space? a). bridge #2-6

- Same question as above, patient also has number #4 and does have decay.

-how would you treatment plan #4 if it were to get an RCT/ what is the most acceptable treatment plan after the RCT?  Take a look at the x-rays and you’ll see decay at/near the level of the alveolar bone.  Choices were A) post and core B) amalgam core C) post and crown and crown D) Crown lengthening, post and core, and crowing

 

-Patient tells you that his anxiety started only in his adult years. Hes 40 something now.  What could be the cause of the anxiety? A) how the dentist approached the patient B) how the front office staff treated him C) His past dental experiences D) the cost of getting dental work done E) people at work telling him their experiences in the dental office

-Look at #31 select what you see A) large buccal lesion B) Occlusal decay C) APP D) something else

 

 

4). Caries small/occlusal (brown spots on pits) on #31 can be seen on a). picture clinically b). xray c). from chart

5). #12 has big caries lesion on mesial up to the bone level. What treatment needed? a). crown lengthening b). post&core – next step because crown only can be on “sound tooth structure” (ferrule rule  = 1mm circular in prep increase strength in …10? Times - check this)

 

Case 3

I think that case was about some man that used to smoke and something about him trying to quit smoking and trying the nicotine patches and other stuff and it not working. Hes only smokes 4-5 cigs a day.

- The best treatment to try to get this patient to stop smoking would be? A) some weird drug B) Nicotine cream C) Nicotine nasal spray D) behavior counseling E)varenciline ( which is a anti smoking drugs but works on the nicotine receptors too )

63 yrs old man … Picture teeth radiograph

1). #14 bone lost mesialy, overhang amalgam. Why? a). forget to put wedge

-what would be the most likely reason to of the amalgam overhang and the cause of the light contact between teeth to teeth? A) using weak amalgam, B)something dumb C) a wedge was not placed when the tooth was restored

2). From #20 big amalgam restoration – you can see on xray pin goes out of tooth distally. Patient complained that floss shred all the time between #20 and #19. What should you do? a). explain patient current situation b). tell pt. that previous dentist performance as bellow standard of dentistry c). extract tooth #20 d). try to cut pin with hand piece

 

3). Can see well defined circular radiolucency under root tips of # 30, #31, #32. Diagnosis? a). PA cyst b). OKC c). ameloblastoma

-same question as above. What seems to be the most likely reason of the radiolucency apical to tooth 31? A) PA cyst B) Odontogenic cyst C) ameloblastoma D) normal anatomy  ok so for this one you need to take a look at the other side of the Pan , and if you do you can see that this RL is in the same location and just doesn’t look anything of the pathology mentioned. it looks just like this! I put normal anatomy cus it’s the submandibular gland fossa.  PS in the Pan on the exam you can also clearly see the mental foramen on both sides.

 

 Case 4

53 yrs old Porcelain-fused-to-metal PFM bridge #8-10. On PAN can see 3rd molars are impacted.

1). Why discoloration of bridge white color/translucency. Every explanation is possible EXECPT? a). metal to thick b). not enough reduction in cervical third of #10  c). opack layer is too much thick

-same question, it asked why the discoloration of the cervical third of the #10. A)not enough labial reduction b)opaque layer too thick C)metal too thick D) something else . Anyways in the clinical picture you can see that the cervical third near the gingiva is clearly more white than the rest of the crown.

2). On clinical picture you can see wear off mandibular incisors. What is a reason for that? a). occlusional habite (bruxism) b). thin dentin/enamel 3). Opposing bridge (reason for that in the next question, if it’s just opposite bridge why all canines are flat?)

 

 

 

-whats the reason for shape of the lower incisor insical thirds? A) thin enamel B) erosion C) oral habit  D) something else

3). Why is that shape of canine – no cusp, flat occlusialy? a). bruxism

-whats the reason for the shape of the canine?in the picture the cusp tip is kinda cut in half. I dunno that looked weird to me but it’s the same idea I went with Bruxism other answer choices didn’t make sense.

4). Should we do 3rd molar extraction for the reason that #1 is close to sinus or #32 is close to mandibular canal?  NO (53yrs, 3rd molars are not bothering him)

5). 3rd molars are #1 disto-bucal and #32 is horizontal angulation of impaction. Plus partial bone coverage. (check in book impaction angulations)  TRUE

-some question about a tooth number 3 needing to be extracted and why separate the tooth when extracting it.

-#3 needs to be extracted, because of its close proximity to the sinus , it is always indicated to separate teeth   It was a 2 part true or false question.  – the tooth was very baldly decayed and there was no clinical crown. Something similar to this minus the anatomical crown on the mesial.

 

 

6). What would be the reason to extract #1?  a). to place implant, if #2 in future would be lost and pt. need a bridge.

7). If you do pulpal thermal test on his posterior teeth you may have Negative/False result. Why? a). Age b). pulpal obliteration/ calcification see xray

 

 

Case 5

On clinical picture you can see adult complete dentition (no missing teeth) in position central incisors touching edge-to-edge. On back, posterior teeth disarticulated.

1). Why discolored pre-molar? Amalgam stain

2). What movement of condyle in TMJ must be for that position? a). rotation b). translation c). both  - for protrusive you do both

-what movement of the condyle is going on in the picture with the lady biting edge to edge? A) both condyles are rotating ( how is that possible? Lol) B) the right condyle rotating while the left translates C) the left condyle rotates while the right condyle translates D) both condyles on translating.

-what kind of position if the patient demonstrating in the edge to edge picture? A) maximum intercuspation B) Centric relation C) Incisor guidance D) Centric position.

 

3). What clinical picture is demonstrating? a). free way space [the space between the max and mand occl surfaces when at physiological rest] b). maximum intercuspation c). central occlusion d).incisor guidance  (I don’t know the answer – I put “a” but may be “b” or “d” also, check it)

4). On xray radiograph you may see circular radiolucency on middle root (close to apex) on #9. Asymptomatic, no pain. Diagnosis? a). lateral periodonal cyst b). radicular cyst c). medial palatine cyst

5). What is the main test needed to be done for diagnosis? a). Thermal vitality test b). EPT c). percussion

 

Case 6

68 year old female wanting to get some work done. Shes taking certain medication but you have to realize that Fosamax is a bisphosphonate! This case mainly tests you on the principles of what you can and cant you to people taking bisphophnates or have an increased risk of ORN.They might switch out the drug name you become familiar with bisphosphonates

68 yrs female came for your appointment with old dentures (both max/mand), that didn’t fit her anymore. She had history of using Fosamax medication (biphosphonate drug to protect bones). She is after cancer surgery, radiation, chemo therapy… On xray all teeth are missing except#6,7,8,9,10 and 25,26,27

1). What is possible diagnosis for her psychotic condition? Depression

-what is the most probable condition that this patient by have? A) Bipolar B) Anxiety C) Depression

-if this patient is on bisphosphonates, which of the following treatments can you render without increased risk of systemic complications A) extraction of all hopeless teeth B) Scale and root planing C) something that you shouldn’t be doing D) RCT

-An expect question? I forget the other choices but the except was that “the patient can proceed to get her extractions done after being off Fosamax for 1 week”

2). If she is after breast cancer chem./radiation + biphosfonate drug Fosamax, what treatment for her you CAN do if needed? a). extraction b). root canal c). alveolar plasty/surgery d). implants  (you can’t touch bone – risk of osteonecrosis)

3). After Fosamax was stoped for 1 week can you do extraction? NO, Fasle

4). What treatment is good for her? Root planning + cleaning, prophy

-what is the most ideal treatment for this patient? Choices were between A)coronal scaling and removable dentures for both max and mand. B) coronal scaling and implant placement in edentulous areas C) Scaling and root planing and something else…I went with this because the patient had moderate to severe bone loss and needing not only coronal and root scaling but also needed root planing on the exposed root surfaces.

 

Case 7

Kid 5 yrs. 9 month fall 3 month ago. Tooth #F fall down. You can see on clinical picture new erupting tooth is appeared. She has a FISTULAR, bump above #E.

1). Tooth #E has luxation. What treatment? Extraction

2). Does age of patient is identical for dental age? Yes, pt.’s age = dental age

3). What would be a treatment? Sealant on all permanent 1st molars

Prophylaxis, fluoride

4). What would be a treatment for posterior crossbite? Bilateral expansion

5). On biteweens you can see small insipient proximal caries on mesial of #19 (between #K and #19) What is a treatment? a). composite b). don’t do anything c). disk between teeth

 

Case 8

This was a Mexican lady that has TMJ problems; I believe she was allergic to penicillin to look out for the premedication question. she cant speak English well and has a loss of hearing from an accident.   In my case the lady did have posterior teeth messing in the in her clinical picture. and there was a picture of the her biting edge to edge .

Mexican female. Has deafness because of accident. Parents help her in transportation and financially. She complain in TMJ pain

1). What would be the easiest to improve? a). OHI oral hygiene b). financial  limitation c). deafness because of accident d). pain from TMJ

-the question was stated differently than above, Mine asked all of these would complicate the treatment of this patient except, A)poor oral care, B)financial limitations C) English problems and deafness of patient D) Pain from TMJ

2). On Xray radiograph you can see #21 is good, normal angulation, no carries, #19 distaly tipped, a big carries lesion, # 14 is supererrupting. What treatment would you recommend? a). build up #19 carries b). build up #21

3). What is next treatment after that? Build up with post and core in only ONE root canal True or False (I don’t know, check)

-when having to build up the molar with you put the coret matrial in one canal the mesial canal (false). Doing this makes the canal stronger (double false)

3). What is LEAST possible when you are upringting #19? a). roots of #19 move facialy b). encorrage of anterior teeth or #21

-what is the least possible movement when up righting #18? A) roots move forward B) anchorage will come from the premolar and anterior teeth C) tooth can extrude and cause occlusal interference

4). When you are upringting #19 what if possible to happend? Occlusial interfearance

5). What is LEAST possible treatment for supererrupting #14? a). Intrusion b). crown c). RCT d). caries txn

6). If you do EXTRUSION of tooth #13? crown-to-root ratio increase and prognosis decrease

 

Case 9

A little girl with CLEFT on clinical picture of Maxillary you can see all teeth lined up normally in ONE line, except #6 & #7, also #10 & #11 are parallel to each other (one behind other).

1). What is reason for strange position of laterals #7 and #10? CLEFT

2). On Cephalometric picture what is LEAST possible diagnosis? a) maxillary prognatism b). class1 c).class2 div 2 d). class3

3). What arrow point on xray?  HYOID

 

Case 10

Man 46 yrs also with CLEFT palate, fixed when he was a kid, by surgery. On clinical picture he has Angular Chelulitis on corner of his mouth. He’s complaining that his dentures are moving and discomfort him and lesion in corner bother him.

1). What is treatment for Angular Chelulitis?  Clotrimasol cream 2%

-the red inflammation that has formed under the maxillary denture is because of a ? A) bacteria B) fungus C) protozoan D) something else 

2). On PAN two opacity left/right  under his mandible?  HYOID

3). He is missing #7 and #10 and bone here (because of cleft) look like resorbed up to10 mm. What would you recommend  treatment?   a). extract #8, #9 and do bridge #6-11  b). saving #8, #9 (not extraction) to preserve a bone/alveolar ridge (not sure – I choose ”b”)

 

 

 

 

 

4). Implants for #7 and

I had a case where I needed to identify the hyoid bone on a pan then find it in the lateral ceph they provided.  This is how it looked. Its basically under the angle of the mandible in a lateral ceph. Don’t get clowned I thought for a second it might be the clavicle lol The other arrows pointed near the vertebrae to confuse you!!

 

 

Sizzle:

Lots of questions were from the 09-10 at least the first 50 pages of the 09-10 document was asked.  The other load of questions were from KAPLAN! PLEASE IF YOU HAVE TIME DO THEM.com(I don’t understand this joke and it sounds stupid)

 

x-rays were basic: they showed you were to look and gave you the description of what you were supposed to identify. Same goes for pictures. I had stuff on white sponge nevus.  and one I had to identify that if the patient had Bell’s palsy, ( they described it in the question and also all you need to remember is that it looks like somebody trying to make a retarded whink!

 

 

Perio:

Flaps on flaps on flaps.  Know these very well, they aren’t too hard but if you just breezed over them you might have a hard time.

Know the signs of inflammation and stuff like that and you should be fine.

 

Pedo:

Calc. Times, and how to treat annoying ass kids. Also some questions on eruption times of course. This section wasn’t too bad. The only hard part was the management of the kids questions other than that it was ok.

 

Pharm:

Basic questions!! This is what you need to look at Kaplan qbank for!! Most of these questions were repeated from there.  Had questions as basic as: why doesn’t LA work if patient has infection, INR is used for what (some options were, to test for penicillin  or some other sort of anti bacterial lol), hardest one that I can remember I got was about the law of mass something… I dunno its in tufts pharm, and the answer is the only one that makes sense though.  If you’ve studied tufts I’m sure you’ll be fine.

 

OS:

This section was a little bit harder that the rest. just remember to look over facial spaces, and extraction sequence and the reasons why you extract them that way.  The 09-10 explanation wasn’t what/how they asked me.  Now  about ORN and when you extract that tooth (pretty straight forward). Most commonly missing tooth, most impacted, and also least likely to be missing, and which would be the most detrimental if it was missing ( loss of arch length type question)

 

Operative:

Lots of questions on composites and the indications of them and whatnot

Stuff on indications of inlays and onlays  (not too specific )

The retention factor of class 1s,2s

Had questions on the C factor

 

Removable:

Wasn’t really hit with material type questions just try to read over fixed from Mosby’s and you’ll do fine in that section.

Lots of denture questions, post. Palatal seal and why its used (obviously not for the mandibular denture like 2 of my options had)

RPD questions weren’t that bad either, they asked stuff such as “ what connects to occlusal rest to the major connector. 

Know stuff about the position of teeth and they sounds they make, asked about this 2 times.

 

Endo:

Very very very basic questions. Know the different Dx.  They give you choices that are obviously not the correct answers. (whens the last time you heard an incipient lesion causing a pulpal necrosis lol) if you’ve reviewed tufts endo file I’m sure you’ll do fine.

 

Pt. MGNT:

The only difficult section in this section were the stupid PPO HMO questions they asked!! And they ask them using definitions and examples not used in Mosby’s. Use a different source other than mosby for that part.

If I were to do anything differently in my last week of studying, I would have read the 09-10 front to back and also read Mosby’s again.

 

 

Questions I got: again I think a lot of mine were from 09-10 so you might want to look at that before your test date.

 

Best way to view maxillary sinus? Water’s view

Question about LED lights, it was an except question…yes they can be powered by battery

 

How can you tell if the infection is of non-odontogenic origin?

 

Referred pain question. Lower mandibular molar was the answer

 

 

 

23.   Pregnant lady? Lay right side up, what is artery are you protecting?

                                Inferior vena cava

26.   Mandibular incisor coming in crowded how do you make space? Interarch distance from primitive space

 

Neurapraxia, the least severe form of peripheral nerve injury, is a contusion of a nerve in which continuity of the epineurial sheath and the axons is maintained

 

 

 

32.   5 year old child having pain what do you give them? Asprin, ibuprofen, codeine, acetominphen

 

36.   OKC-most likely to reoccur

 

37.   Nevoid BC

Nevoid basal cell carcinoma syndrome (Gorlin syndrome) is an autosomal dominant inherited condition that exhibits high penetrance and variable expressivity. The syndrome is caused by mutations in patched (PTCH), a tumor suppressor gene that has been mapped to chromosome 9q22.3-q31. Approximately 35% to 50% of affected patients represent new mutations. The chief components are multiple basal cell carcinomas of the skin, odontogenic keratocysts, intracranial calcification, and rib and vertebral anomalies. Many other anomalies have been reported in these patients and probably also represent manifestations of the syndrome. The prevalence of Gorlin syndrome is estimated to be about 1 in 60,000.

 

 

39.   Material least to do impression with-irreverisble hydrocolloid

Polyether is the worst p334 (mosby )

40.   H2 histamine-gastric reflux

H2 inhibit the action of histamine in the stomach and reduce stomach acids.
ex. Rantidine, cimetidine, famotidine.

 

45.   Calcified canal what do you do-refer

46.   When is it ok to do a temporary fixing on patient? Emergency

47.   Incidence 100/1000

         Incidence: indicates the number of new cases that will occur within a population over a period of time (e.g., the incidence of people dying of oral cancer is 10% per year in men aged 55 to 59 in our community).

 

How do you get a child acting out to act favorably? Let them watch another child behaving-Modeling  technique.

 

54.   Class 3- cleft palate, cleft lip

 

59.   Bevel for occlusal on a crown? structural integrity

 

68.   Amoxicillin and clonavonic acid is combined to keep from degrading beta lactam ring

75.   Why do you take denture out at night

Patients should be told that dentures must be left out of the mouth at night to provide needed rest from the stresses they create on the residual ridges

72.   If patient wants to last for 8 hours which is long acting drug? Aspirin, ibuprofen, acetominaphine, n-something

 

82.   ANUG comes with spirochetes

 

83.   Pic of white spongy nevus- the clue on the description was plaques!!

                White sponge nevus: of buccal mucosa.

The lesions of white sponge nevus usually appear at birth or in early childhood, but sometimes the condition develops during adolescence. Symmetrical, thickened, white, corrugated or velvety, diffuse plaques affect the buccal mucosa bilaterally in most instances

 

88.   How long do you splint with avulsion. 7-10 days,

91.   How much do you take off facial for veneer? .5-1m

90.   Ppm in water-1

1 ppm = 1 gm/L

95.   Contraindication for diazepam-diabetic, pregnancy, etc

97.   Ging recession 5-6mm on #4 & 20, Hemoglobin of 12. Wht do you do? Treat, refer to dr, scaling n root planning

Hemoglobin (male)

13.5-17.5 g/dL

Hemoglobin (female)

12.3-15.3 g/dL

98.   What muscle covers denture? Buccinators, masseter, lat & med pterygoid

99.   What provides lingual retention? Mylohyoid

100.  Neurofibromatosis- axiallary freckling, café- au-late, lesch nodules

101.                        Most impacted tooth? Mx k9

108.                        Base metal vs noble metal-single crown-3 unit bridge

103.                        Purpose of hex implant :

 in an internal hex implant, the antirotational feature

104.                        Push on rest seat it comes up? Base doesn’t come up bc of resin

105.                        2nd to s. mutan-L. bacillus

109.                        Papillon le fever

110.                        Oligiodontia-ectodermal dysplasia

111.                        Collimation-tube

A collimator is a metallic barrier with an aperture in the middle used to reduce the size of the x-ray beam

112.                        Erosion- bullemia

113.                        Patient gets 25% home bleaching. Wrong its 10% but 2nd part is true

The current home bleaching technique, employing a custom-fit tray containing 10% carbamide peroxide solution, was first used by Klusmier in the late 1960s.6 In-office bleaching materials are usually supplied in concentrations of 35% hydrogen peroxide, although some concentrations may be as high as 50%. The caustic nature of 35% to 50% hydrogen peroxide mandates that the soft tissues be isolated from any possible contact with the bleaching material

114.                        What goes into cavernous sinus from upper lip? Subcutaneous tissue

115.                        URI-no NO2

116.                        In posterior composite why do you have to redo-occlusal-wear

117.                        Periosteum-sharpeys fibers, cementum, alveolar bone, or all 3

118.                        Symphisis-intraocciptal, sphenoocciptal, which bone forms last

122.                        Nonworking-bull working-lubl

123.                        Transillumination-vertical fracture

124.                        Minor connector connects to

127.                        To far superior and anterior dentures-what sounds

The labiodental sounds f and v

128.                        If you did  a DO what axioline angle is not there

129.                        If you fall and break incisor which class is it due to? Class 2 div 1

133.                        Support area for max and mand denture

Maxilla: residual ridge primary, rugea secondary

Mandible: buccal shelf primary

134.                        Cleidocranial dysplasia-supernumary teeth

136.                        Nausea and vomiting from opoid receptor poisoning? Chemoreceptor trigger zone

138.                        PCN and tetracycline cancels each other out

1.       Guy has problem with a tooth and has a hole drilled thru the O of MOD composite and the pain is relieved. What caused it? polymerization shrinkage

 

8.       Sialolith commonly found?  Submandibular gland-wharton’s duct

6.       Neuropraxia question-nothing severed, perioneum intact, can get it from stretching.

11.   What is best way to view TMJ? MRI

27.   Grand mal(tonic-clonic) seizure drug of choice? Dilantin( phenytoin)

25.   What is malignant? 25.   What is malignant? Fibrous dysplasia, pagent’s , central giant cell granuloma???

 

32.   What do you do with probe if furcation is wide and narrow, narrow, wide? Probe or cant probe? Grade 1 probe goes less than 1/3, G2 probe goes more than 1mm(do GTR n graft), G3 probe goes straight thru

Grade I is incipient bone loss, grade II is partial bone loss (cul-de-sac), and grade III is total bone loss with through-and-through opening of the furcation. Grade IV is similar to grade III, but with gingival recession exposing the furcation to view.

(Newman, Michael G.. Carranza's Clinical Periodontology, 10th Edition. Saunders Book Company, 072006. 28.5.8).

33.   What do you do for a furcation that you can see through? T or F. Tunneling, GTR membrane?

Class I: Early Defects

Incipient or early furcation defects (class I) are amenable to conservative periodontal therapy. Because the pocket is suprabony and has not entered the furcation, oral hygiene, scaling, and root planing are effective.15 Any thick overhanging margins of restorations, facial grooves, or CEPs should be eliminated by odontoplasty, recontouring, or replacement. The resolution of inflammation and subsequent repair of the periodontal ligament and bone are usually sufficient to restore periodontal health.

Class II

Once a horizontal component to the furcation has developed (class II), therapy becomes more complicated. Shallow horizontal involvement without significant vertical bone loss usually responds favorably to localized flap procedures with odontoplasty and osteoplasty. Isolated deep class II furcations may respond to flap procedures with osteoplasty and odontoplasty. This reduces the dome of the furcation and alters gingival contours to facilitate the patient’s plaque removal.

Classes II to IV: Advanced Defects

The development of a significant horizontal component to one or more furcations of a multirooted tooth (late class II, class III or IV13) or the development of a deep vertical component to the furca poses additional problems. Nonsurgical treatment is usually ineffective because the ability to instrument the tooth surfaces adequately is compromised.30,36 Periodontal surgery, endodontic therapy, and restoration of the tooth may be required to retain the tooth.

 

34.   8 year old Central incisor canal is constricted but has apical RL what do you do? Refer

35.   What is worst if doing a RCT? Insufficient obturation, insufficient cleaning and shaping,

36.   In RCT was is plastic post good to use? Same strength as dentin, better strength then steel, same strength as steel, when cemented you can view on xray

37.   RCT done and years have RL below what caused this? actinomyces

38.   Xray of woman who had molar extracted, now has infection, what caused this? Osteomyletis, residual cyst

A residual cyst is a cyst that remains after incomplete removal of the original cyst. The term residual is used most often for a radicular cyst that may be left behind, most commonly after extraction of a tooth.

39.   C factor(configuration factor)- composite ratio for bonded to unbounded

The C-factor is related to the cavity preparation geometry and is represented by the ratio of bonded to nonbonded surface areas. Residual polymerization stress increases directly with this ratio.

40.   Bilateral split osteotomy what nerve do you worry about severing? Inferior alveolar

41.   Cleft palate/lip- class 3

42.   Main reason for redoing anterior composite-discolored


43.   Fluoride- how much do we use in community water 0.7-1.2 ppm

78.   Fractured mandible how long is appropriate to keep in closed reduction? 4weeks, 6 weeks,

 

 

Winter2011

 

1.       Which test is MOST valuable in an tooth that needs testing that has an open apex…young tooth = cold test (EPT does NOT work)

2.       Internal bleaching will MOST likely cause? Extracanal cervical resorption, replacement resorption, external resorption (idk? Was not aware of what “cervical” resorption was especially when given extracanal cervical resorption AND external  resorption in answer choices???)

3.       Apexification-done when tooth is NOT vital (aka need RCT) and you need to close the apex so you can get a seal for the gutta percha; Apexiogenesis-done when tooth IS VITAL and RCT is NOT necessary no matter what has happened (but you have to know the situations in which a tooth would be vital or nonvital)

4.       When tooth is traumatically intruded… LET IT ERUPT!

5.       Difference between reversible and irreversible and necrotic SYMPTOMS… how long pain lingers to COLD test etc.

6.       Sensitivity to percussion and biting you know you have acute apical periodontitis

7.       MOST likely cause of pulpal inflammation= DECAY/BACTERIA/CARIES etc.

8.       Pain from which one, mandibular premolar or mandibular molar, refers to the ear? Idk? I have a hard time choosing between the two

9.       SLOB rule question, Buccal root shot from M, now shoot from D and its oppositie the lingual root shot from mesial blah blah blah (SAME LINGUAL OPPOSITE BUCCAL)

10.   Vertical root fracture= EXTRACT

11.   Disadvantage of NaOCl: toxic to tissues (does NOT remove smear layer btw)

12.   Difference b/w self etch and total etch: self etch does NOT remove smear layer

13.   Bacteria responsible for pulpal infection: not specifically which one but answer choice was one or more than one bacteria? Idk

14.   Ledge… bypass the ledge

15.   Tooth most likely to have 2 canals: max 1st pm

16.   Where canals are in mand molar most likely: 3= 2 M and 1D

17.   Which access preparation canals look like a C shape? I forget mand molar?

18.   What facilitates RCT… NOT calcified canals

19.   pH 5.5 critical

20.   treatment sequencing Mosby p. 38

21.   problem with amalgam lies in environmental disposal

22.   difference b/w resistance and retention forms

23.   knowing that liners and bases and recurrent decay can all appear radiolucent

24.   on a pan 2 bones that can appear below mandible= hyoids

25.   place calcium hydroxide then resin glass ionomer base then restoration

26.   admix= spherical and boxy particles

27.   breakdown of a composite margin most likely due to: shrinkage during curing, expansion, or overfinished margins? Idk

28.   large posterior decay: large composite is NOT ideal

29.   COTE = gold< amal< unfilled composite

30.   Alveolar osteitis= dry socket

31.   Bleeding after ext 3 hours later à remove clot to locate location of problem

32.   Pt has 3rd molars… what do you do: tell the patient 3rd molars cause crowding and need to come out, do not inform patient about them, 3rd molars are associated with cysts and various pathologies and need to be extracted, or do not do anything about them

33.   Pt has cirrhosis of liver: what is his liver mostly composed of? Hepatocytes, fibroblasts, hematopoetic cells… idk

34.   Pan had a 3rd molar that was basically straight up and down maybe tilted to distal by 2 mm but erupted fully… it was NOT distoangular impaction (know what these look like on an xray-distoangular, mesioangular impaction)

35.   Day after ext pt comes back with fever and sick feeling… give him different antibiotic (bc he has AA bacterial infection which is associated with ext is what I thought) OR refer him to proper specialist… idk

36.   Adult pt has crossbite needs fixin: listed 3 ortho appliances-even quad helix (tricky), or surgery- adult so I chose surgery

37.   Distraction osteogeneis good for LARGE movements

38.   ANB 6… Class 2 dental or Class 2 skeletal malformation?

39.   Trigeminal neuralgia is NOT associated with a spontaneous dull ache

40.   Pt reports 3rd molars have been ext years ago… pan reveals small round RO in area of #17: osteitis

41.   Pt has cirrhosis of liver: what is best: lidocaine, mepivicaine, bupivicaine, articaine

42.   Infected tissue… LA will be in ionized form

43.   Prilocaine- methemoglobinemia

44.   Swelling in front of SCM:

45.   Cleft lip/palate 1:700

46.   First ‘dental formation’: (weird!) 6 weeks, 16 weeks, 32 weeks idk (teeth, palate, what idk what you’re talking about…)

47.   Syphilis lesion that looks most like herpes? Idk

48.   Pemphigoid against “basement membrane” (pemphigus against something else)

49.   PCOD ant aa female ANT MANDIBLE

50.   Xray teeth with no pulps: DI pg 124 mosby

51.   Target made of tungsten

52.   Mosby 141 post wall of zygomatic process on xray

53.   Man w/ ill fitting partials…indurated ulcerated lesion lat border of tongue: SCC or trauma from partial? SCC and biopsy

54.   Untreated decay mostly in AA… oral cancer mostly in AA males mosby 207

55.   Pg 163, 164, 165 know like back of your hand… quad helix corrects crossbites, use these all in GROWING ppl, surgery for adult…when to use LLA or band loop PLEASE KNOW THIS! LLA-bilateral loss, #19 and 30 NOT erupted yet= need distal shoe… if they are erupted need BL

56.   Chi square= categories, ttest=averages or means

57.   Modeling pg 225 mosbys

58.   Probing + recession= clinical attachment level

59.   Do NOT attempt perio surgery until you have tried and failed at initial SRP therapy

60.   45-90 angle on SRP instruments

61.   Most cost effective: stress oral hygiene home care

62.   Pg 272 intrinsic activity and maximal effect and efficacy and receptors and affinity just know it all and how its all interrelated

63.   Beta blockers end in –olol, anti GERD drugs –prazoles (omniprazole=tagament/prevacid etc)

64.   Pharm: carbamazipene, atropine, mechanism of tricyclic antidepressants

65.   If you change vertical dimension occlusion during fabricating a complete denture what do you have to REDO? CR or facebow… CR!

66.   How to help pt who gags with their denture? Tell em to put the denture in for as long as they can, put a spoon and hold it for as long as they can tolerate it-do this over and over-YES

67.   Implants 3mm apart

68.   323-324 mosby denture phonetics… just know why you make all the sounds ALL of them, esp f,v, ph (do it yourself and you’ll remember)

69.   Kennedy classifications must know cold

70.   Value is lightness or darkness

71.   Primary tooth with most buccal and lingual convergence…idk

72.   Hemoglobin type in sickle cell disease: A, C, F, or S? idk S

73.   Pan with laterals missing in photograph but present on pan just impacted, 9 years old, canines impacted… (#1 dental age conincides with his chronological age-NO #2 dental anomaly in this kid occurs more in permanent than primary dentition, this occurs in proliferation stage-TRUE OR FALSE idk)

74.   Pan with 2, 14, 15 w/ 50% bone loss and bombed out 14 and 2 and unilateral edentulous area on mand… tx plan? A.crown lengthening and crowns B.orthodontic tx on maxillary C.max class 2 partial D. MAND CLASS 2 PARTIAL

75.   Radiolucency on pm with MO amalgam could be all EXCEPT: recurrent decay, liner, base, something off the wall…

76.   “your fees are high!” your response should be? “it seems you are concerned if getting your dental care is worth it?” OR “ my fees are the same as all the other dentists” or “I have to pay bills” blah blah

77.   Modeling… use sister or other kid to show uncooperative kid what to do

78.   Reason for everything: PLAQUE

 

LaJoy

Well very interesting is all I can say about this test. From what I was given, the only advice I can offer is dat Remembered Questions are GOLDEN.com!!! lol!

Make sure you do as many as you can find. And try to look up the answers to the ones that don’t have definitive answers. The 09-10 is helpful also if you start on it about two weeks before your test and get through it all. Then the last week just do remembered questions…remembered questions…and then do some more remembered questions! Lolol

I tried my best to remember as many new questions or variations of the questions that have already been posted as I could. The ans choices are either the ones I kept going back and forth b2wn or the only ones I could remember so double check the validity..lol… GOOD LUCK CLASSMATES! J

 

  1. Mandibular division of the trigeminal nerve exits the skull from what foramen? Foramen ovale, Superior Orbital fissure
  2. How is fluorosis classified? Amount ingested, #of surfaces fluorosis divided by the # of total teeth, # of teeth with fluorosis compared with the # not
  3. Effective dose vs. Absorbed dose
  4. Distance is 12ft instead of 4ft. Calculate the distance from the source
  5. Clinical attachment lost is? CEJ to the bottom of the pocket, gingival margin to the bottom of the pocket
  6. Metal denture base (Which is not an advantage)? Thermal conductivity, weight on the maxillary arch
  7. What does IgG do? Bind to the host antigen, affect IgE and mast cells
  8. All the walls are missing except the distal wall which is present. What type of defect is it?
  9. Distal wedge
  10. Gingivitis in puberty and pregnancy caused by? Plaque, estradiol ( most potent for sexual maturity ), estrogen, or progesterone
  11. What structure is crushed when preg. Woman is in the SUPINE position? IVC, abdominal aorta
  12. Picture of “bump on the gums” and pt is NOT pregnant: Parulis, Pyogenic Granuloma, Peripheral Giant Cell Granulom

(parulis: subperiosteal abcess of the gum)

  1. What has histology similar to congenital epulis: granular cell tumor, peripheral ossifying fibroma
  2. Which has the characteristic to have carcinoma in situ? Leukoplakia, Erythroplakia
  3. Where do you see increase in alkaline phosphatase? Hyperparathyroidism
  4. Punched out radiouluncy? Multiple myloma
  5. RL in the midline of the palate of vital teeth b2wn the max CI: nasopalatine duct, median palatal cyst
  6. A pic of a RL around an impacted 2nd Mand PM and the primary 2nd Molar sitting on top of it? Dentigerous cyst
  7.  A lesion that malignancy is suspected wht should you do? Needle aspirate, incisional biopsy
  8. Pic of a erythemous lesion on the middle of the tongue and description says it is also on the palate? Syphilis, gonorrhea
  9. Bilateral red erythemous lesion? Erosive lichen planus
  10. Disease to know/look up: Sturge Weber, Lymphangioma, Fibroma, Herpangia, Actinic Keratosis
  11. Which would you not see a radiographic difference? Dentin dysplasia, enamel hypoplasia, AI
  12. Pic of gingival hyperplasia? Fibroma, Fibromatosis
  13. Slaviary gland obstruction of the partiod (Stendon’s) gland?
  14. Which of the following is the last sense to leave after LA administration: pain, temp, touch, pressure
  15. Osteoradionecrosis of the jaw is likely seen in? 2300 rads of radiation therapy, IV bisphonates for a year, chemotherapy
  16. The speno-occiptal synchondrosis is: a suture, cartilage, intramembraneous
  17. Gov’r regulation in 1997 that mandated…? OSHA, HIPPA
  18. Dentist file insurance for post and core buildup and crown. Insurance says that they should be charged 2getha and only reimburses for the crown. What is this an ex of? Downcoding, upcoding, bundling, unbundling
  19. Why is polyether not a good impression material (disadvantages)? Get stuck in the pt mouth, low solubility
  20. Dentist is doing a MOD composite restoration. Pt complains of pain 3 days after. What is the reason? (polymerization shrinkage was NOT an answer…) pressure from expansion
  21. Advantage of indirect comp?

(advantages of indirect comp frm Kaplan: less polymerization shrinkage, better proximal contact, less marginal leakage, greater strength, less post-op pain)

  1. Which drug is NOT used to tx Angina? Nitrates and nitrites,
  2. Which drug does NOT play a role in platelet funx? IB profen, Asprin, Ginseng, Plavix
  3. Digoxin assoc with kidney funx?
  4. Which is true of pseudomembranous colitis? Due to overgrowth of clostridium difficle, drug of choice to tx it is metronizadole

 

As far as the case questions, they weren’t so bad. Acutally kindof fun…made you feel like a forreal dentist (lol!) which is also a bit scary b/c wht you think is a good tx ADA might not agree. So the best thing I can say abt that is go over your ortho…had A LOT of ortho (bimax protrusion, class I, Class II, profiles, if the ANB angle is 6 what type of profile does this pt have, and lateral ceph radiographs). A couple of Endo diagnosis, and a lot of prophylaxis, like should pt be premedicated n if so wht to give (so knw reasons for premedication…endocarditis and total joint replacement..and the dosage)

 

Jeweled

 

Hey folks!!! If you haven't opened up the 09-10 file OPEN IT!!!! LEARN IT!!!! KNOW IT!!!! But most importantly UNDERSTAND IT!!!!!! If I were you I would also look at any other files related to 09-10.

 

My test had a lot of ENDO, it wasn't hard but just be clear abt ACUTE PERIRADICULAR ABSCESS vs ACUTE PERIODONTAL PERIODONTITIS (I think I wrote those correctly...) 

 

I had about 5 hue, chroma, value questions...I can't remember the questions but if I do I'll post again. 

 

Oral surgery:

 What is the ext sequence for molar ext 321 to protect the tuberosity, 123 to protect the tuberosity, 213, 321 etc, etc

What is the most common complication after ext? dry socket, infx, hemmorhage, something else can't remember

what is a complication with bilateral split osteotomy? Damage to the IAN.

If a pt has been taking corticosteroids for a long time what do you have to be concerned with? Adrenal insufficiency

If a pt has been taking bisphosphonates they may be at risk for osteonecrosis b/c: they have to have radiation therapy (something like that)

If a pt is going to have head and neck radiation what should be done? the pt should have questionable teeth extracted prior to radiation

 

Fluoride:

SO THAT YOU WON'T GET CONFUSED I WOULD RIGHT DOWN THE SUPPLEMENTATION CHART BEFORE YOU START THE TEST.

What is the supplementation for a 5yr old in a community with 0.28ppm fluoride?

What is the supplementation for a 7yr old(??) if the community water is 0.75ppm?

 

Pedo:

When is calcification for a max central incisor?

 

Ortho:

If a pt has had MENARCHE (her PERIOD aka menstral cycle) what does this say about her growth spurt? I put it is AT the PEAK of the growth spurt ( the other choices were before or after the growth spurt)

 

Fixed:

Which cantilever has the LEAST success: I put a LATERAL abutment with a pontic replacing a central.

Base nobles are used for: long span FPDs

Why does an FPD keep failing? I put occlusal interferances

 

CD:

What causes angular cheilitis?

What is a sign of success for a CD? good peripheral seal

 

Pt management:

The ADA covers all except: Licensure

Modeling

Replacing amalgams with composites b/c of "allergy" this is covered under the which ADA code: Veracity

Which is a controlled stimulus in the dental office? fear, anxiety, DENTAL CHAIR (that's the only one that made sense)

I had an incidence question: the answer was 100/1000 but I had to read the question a couple times b/c of the way it was worded so BE CAREFUL.

I had some other pt mgmt questions but they weren't really hard...but there were terms that I wasn't familiar with but just read carefully and make educated answer choices.

 

PICS:

Bell's Palsy

Pterygomaxillary fissue

Nutrient canal

Basal cell carcinoma

RL under the inf alveolar canal...but it wasn't called STAFNE's i put salivary duct something that's the only one that made sense and I THINK (I haven't looked it up) but it maybe another name for stafne's bone defect...??

 

 

Oral Path:

Lisch nodules, axillary pigmentation...Neurofibromatosis aka von reckinghausen

highest recurrent rate: OKC

highest risk of malignancy: i put osteomas...?

 

Ortho:

After ortho why does rotation occur again: apical fibers, oblique fibers, neural something, TRANSCEPTAL FIBERS was my choice

 

Pharm:

What is the reversal for diazepam (benzodiapine): Flumazenil

If a pt wants pain therapy for 8hrs what do you give? naproxen

Pain med for 5yo? acetoaminophen

What should you avoid with ginseng? warfarin..?

INR is a test for what (I had this question twice, it was asked 2 diff ways)

 

Implants:

Distance between implants: 3mm

what temp will cause necrosis after 1-5min?: 57,43,47 deg Celcius

what is the LEAST likely cause of necrosis?: HIGH TORQUE

 

 

Ok so this is all I can remember off the top of my head. If I think of some more I'll repost.

 

~A!

Oh and LAST BUT NOT LEAST PRAY and BELIEVE!!!!!!

 

Post a Comment

Previous Post Next Post

Subscribe

Get email notifications

Contact Form